Метод гаусса решение линейных систем уравнений: Онлайн калькулятор. Решение систем линейных уравнений. Метод Гаусса.

Содержание

Решение системы линейных уравнений. Метод Гаусса

Решение системы линейных уравнений. Метод Гаусса

Метод Гаусса — это способ решения системы линейных алгебраических уравнений (СЛАУ), который состоит в постепенном понижении порядка системы и исключении неизвестных.

Решение СЛАУ методом Гаусса состоит из двух этапов:

  1. На первом этапе выполняется «прямой ход», когда путём простых преобразований над строками систему приводят к ступенчатой или треугольной форме, либо устанавливают, что система несовместна. Среди элементов первого столбца матрицы отбирают ненулевой, перемещают его в крайнее верхнее положение путем перестановки строк и вычитают получившуюся после перестановки первую строку из оставшихся строк, домножив её на величину, равную отношению первого элемента каждой из этих строк к первому элементу первой строки, обнуляя тем самым столбец под ним.

    Далее первую строку и первый столбец мысленно вычеркивают. Процесс повторяют пока не останется матрица нулевого размера. Если на какой-то из итераций среди элементов первого столбца не найден ненулевой, то переходят к следующему столбцу и выполняют аналогичную операцию.

  2. На втором этапе выполняется «обратный ход». Его суть в том, чтобы выразить все получившиеся базисные переменные через небазисные и построить фундаментальную систему решений. Если все переменные являются базисными, то выразить в численном виде единственное решение системы линейных уравнений. Эта процедура начинается с последнего уравнения. Из него выражают соответствующую единственную базисную переменную и подставляют в предыдущие уравнения. Затем так продолжают далее, поднимаясь по «ступенькам» наверх. Каждой строчке соответствует только одна базисная переменная, поэтому на каждом шаге, кроме последнего (самого верхнего), ситуация повторяет случай последней строки.

Пусть есть исходная система, которая выглядит следующим образом:

(1)

Матрица A называется основной матрицей системы, а матрица b — столбцом свободных членов.

Согласно свойству элементарных преобразований над строками основную матрицу этой системы можно привести к ступенчатому виду. Эти же преобразования требуется применять к столбцу свободных членов:

При этом считаем, что базисный минор (ненулевой минор максимального порядка) основной матрицы находится в верхнем левом углу, т.е. в него входят только коэффициенты при переменных xj1, …, xjr. Такого расположения минора можно добиться путем перестановки столбцов основной матрицы и соответствующей перенумерацией переменных.

Таким образом, переменные xj1, …, xjr называются главными переменными. Все остальные называются свободными.

Если хотя бы одно число βi ≠ 0, где i > r, то рассматриваемая система несовместна.

Пусть, что βi = 0 для любых i > r.

Перенесём свободные переменные за знаки равенств и поделим каждое из уравнений системы на свой коэффициент при самом левом  (αij, i = 1, …, r, где i — номер строки):

(2)

Где i = 1, …, r, k = i + 1, …, n.

Если свободным переменным системы (2) придавать все возможные значения и решать новую систему относительно главных неизвестных снизу вверх (т.е. от нижнего уравнения к верхнему), то в результате получим все решения этой СЛАУ. Так как эта система найдена путём элементарных преобразований над исходной системой (1), то по теореме об эквивалентности при элементарных преобразованиях системы (1) и (2) эквивалентны, т.е. множества их решений совпадают.

Следствия:

  • Если в совместной системе все переменные главные, то данная система является определённой.

  • Если количество переменных в системе превосходит число уравнений, то данная система является либо неопределённой, либо несовместной.

См. также:

Библиотека методов и моделей | ISmLinearEquations

Решение методом Гаусса СЛАУ 3-5-ого порядка

Метод Гаусса решения систем линейных алгебраических уравнений состоит в последовательном исключении неизвестных с помощью элементарных преобразований и сведении к верхней треугольной (ступенчатой или трапециевидной). После чего решают систему с конца к началу, подстановкой найденных решений.

Рассмотрим примеры решения систем линейных уравнений методом Гаусса, взяв за справочник сборник задач Дубовика В.П., Юрика И.И. «Высшая математика».

————-

Задача.

Решить систему линейных алгебраических уравнений.

1) (1. 189)

2) (4. 195)

3) (4. 198)

Решение.

1) Преобразуем исходную систему к ступенчатому виду. Для этого от второго уравнения вычтем первое, умноженное на 3, а от четвертого вычтем первое, умноженное на 4.

В результате с третьего уравнения имеем Полученное значение подставляем в исходное уравнение для нахождения

Полученные значения подставляем в первое уравнение

Решением системы трех линейных уравнений будут следующие значения переменных

2) Имеем систему трех уравнений с четырьмя неизвестными. В таких случаях одна переменная может быть свободна, а остальные будут выражаться через нее. Сведем систему к ступенчатому виду. Для этого от второго и третьего уравнения вычтем первое

Из последних двух уравнений получаем идентичные решения

.

После подстановки в первое уравнение получим

Данное уравнение связывает три переменные. Таким образом любая из переменных может быть выражена через две других

Итак получим следующее решение

3) Имеем разреженную систему линейных уравнений пятого порядка с пятью неизвестными. Сведем ее к ступенчатому виду. От второго уравнения вычтем первое и запишем в удобном для анализа виде

Из второго уравнения находим, что . Подставляем значения во все нижние уравнения и переносим за знак равенства. Также поменяем второе с третьим уравнения местами

Четвёртое и пятое уравнения эквивалентны. Выразим одну из переменных через другую

Полученное значение подставим во второе уравнение и найдем

Из первого уравнения определяем

Решение системы уравнений следующее

При вычислениях систем линейных алгебраических уравнений методом Гаусса нужно свезти систему линейных уравнений к ступенчатому виду. Для этого удобно записывать переменные под переменными, как в последнем примере, это ускорит решение. Остальное все зависит от матрицы, которую нужно решить и Ваших умений.

———————————————-

Посмотреть материалы:

Решение систем линейных уравнений методом Гаусса

Теория

Классическим методом решения систем линейных алгебраических уравнений является метод Гаусса (метод исключений Гаусса). Суть метода — это последовательное исключение неизвестных, т.е. когда с помощью элементарных преобразований система уравнений приводится к равносильной системе ступенчатого вида, из которой последовательно, начиная с последних переменных, находятся все остальные переменные.

Матрица, составленная из все ai,j, называется основной матрицей системы. Если к этой матрице добавить вектор столбец, составленный из bi, то такая матрица называется расширенной матрицей системы.

Теорема Кронекера-Капелли (условие совместности системы): системат совместна тогда и только тогда, ранг ее основной матрицы равен рангу ее расширенной матрицы.

Алгоритм решения СЛАУ методом Гаусса подразделяется на два этапа:

  • На первом этапе (прямой ход) система приводится ступенчатой или треугольной форме. Вычтем из второго уравнения системы первое, умноженное на такое число, чтобы обнулился коэффициент при x1. Затем таким же образом вычтем первое уравнение из третьего, четвертого и т.д. Тогда исключаются все коэффициенты первого столбца, лежащие ниже главной диагонали. Затем при помощи второго уравнения исключим из третьего, четвертого и т.д. уравнений коэффициенты второго столбца. Последовательно продолжая этот процесс, исключим из матрицы все коэффициенты, лежащие ниже главной даигонали.
  • На втором этапе (обратный ход) выражаем все получившиеся базисные переменные через небазисные и построим фундаментальную систему решений. Если все переменные являются базисными, то получим единственное решение системы линейных уравнений. Эта процедура начинается с последнего уравнения, из которого выражают соответствующую базисную переменную (а она там всего одна) и подставляют в предыдущие уравнения, и так далее, поднимаясь по «ступенькам» наверх. Каждой строчке соответствует ровно одна базисная переменная, поэтому на каждом шаге, кроме последнего (самого верхнего), ситуация в точности повторяет случай последней строки.

Решение системы линейных уравнений методом Гаусса в MS Excel

На днях понадобилось найти корни системы линейных уравнений методом Гаусса в Microsoft Excel. Готовый алгоритм решения можно найти в книге Гарнаева «Использование Excel и VBA в экономике и финансах», но объяснение там очень скудное и не совсем понятное. Постараюсь описать подробней для тех, кому может понадобиться этот алгоритм.

Лирическое отступление: в тексте будет предлагаться ввести в диапазон ячеек формулу вида: {=A1:B3+$C$2:$C$3} и т.п., это так-называемые «формулы массива» (формула, выполняющая несколько вычислений над одним или несколькими наборами значений, а затем возвращающая один или несколько результатов. Формулы массива заключены в фигурные скобки { }). Microsoft Excel автоматически заключает ее в фигурные скобки ( { } ). Для введения такого типа формул необходимо выделить весь диапазон, куда нужно вставить формулу, в первой ячейке ввести формулу без фигурных скобок (для примера выше — =A1:B3+$C$2:$C$3) и нажать Ctrl+Shift+Enter.

Пускай имеем систему линейных уравнений:

1. Запишем коэффициенты системы уравнений в ячейки A1:D4 а столбец свободных членов в ячейки

E1:E4. Если в ячейке A1 находится 0, необходимо поменять строки местами так, чтоб в этой ячейке было отличное от ноля значение. Для большей наглядности можно добавить заливку ячеек, в которых находятся свободные члены.

2. Необходимо коэффициент при x1 во всех уравнениях кроме первого привести к 0. Для начала сделаем это для второго уравнения. Скопируем первую строку в ячейки A6:E6 без изменений, в ячейки A7:E7 необходимо ввести формулу: {=A2:E2-$A$1:$E$1*(A2/$A$1)}. Таким образом мы от второй строки отнимаем первую, умноженную на A2/$A$1, т.е. отношение первых коэффициентов второго и первого уравнения. Для удобства заполнения строк 8 и 9 ссылки на ячейки первой строки необходимо использовать абсолютные (используем символ $).

3. Копируем введенную формулу формулу в строки 8 и 9, таким образом избавляемся от коэффициентов перед x1 во всех уравнениях кроме первого.

4. Теперь приведем коэффициенты перед x2 в третьем и четвертом уравнении к 0. Для этого скопируем полученные 6-ю и 7-ю строки (только значения) в строки 11 и 12, а в ячейки A13:E13 введем формулу {=A8:E8-$A$7:$E$7*(B8/$B$7)}, которую затем скопируем в ячейки A14:E14. Таким образом реализуется разность строк 8 и 7, умноженных на коэффициент B8/$B$7. Не забываем проводить перестановку строк, чтоб избавиться от 0 в знаменателе дроби.

5. Осталось привести коэффициент при x3 в четвертом уравнении к 0, для этого вновь проделаем аналогичные действия: скопируем полученные 11, 12 и 13-ю строки (только значения) в строки 16-18, а в ячейки A19:E19 введем формулу {=A14:E14-$A$13:$E$13*(C14/$C$13)}

. Таким образом реализуется разность строк 14 и 13, умноженных на коэффициент C14/$C$13. Не забываем проводить перестановку строк, чтоб избавиться от 0 в знаменателе дроби.

6. Прямая прогонка методом Гаусса завершена. Обратную прогонку начнем с последней строки полученной матрицы. Необходимо все элементы последней строки разделить на коэффициент при x4. Для этого в строку 24 введем формулу {=A19:E19/D19}.

7. Приведем все строки к подобному виду, для этого заполним строки 23, 22, 21 следующими формулами:
23: {=(A18:E18-A24:E24*D18)/C18} — отнимаем от третьей строки четвертую умноженную на коэффициент при x4 третьей строки.
22: {=(A17:E17-A23:E23*C17-A24:E24*D17)/B17} — от второй строки отнимаем третью и четвертую, умноженные на соответствующие коэффициенты.
21: {=(A16:E16-A22:E22*B16-A23:E23*C16-A24:E24*D16)/A16} — от первой строки отнимаем вторую, третью и четвертую, умноженные на соответствующие коэффициенты.
Результат (корни уравнения) вычислены в ячейках E21:E24.

UPDATE от 25 апреля 2012 г. Выкладываю xls-файл с решением линейных уравнений методом Гаусса в Microsoft Excel:

Решающих систем с исключением Гаусса — College Algebra

Цели обучения

В этом разделе вы:

  • Запишите расширенную матрицу системы уравнений.
  • Напишите систему уравнений из расширенной матрицы.
  • Выполняет операции со строками в матрице.
  • Решите систему линейных уравнений с помощью матриц.

Немецкий математик Карл Фридрих Гаусс (1777–1855).

Карл Фридрих Гаусс жил в конце 18 — начале 19 века, но он по-прежнему считается одним из самых плодовитых математиков в истории.Его вклад в математику и физику охватывает такие области, как алгебра, теория чисел, анализ, дифференциальная геометрия, астрономия и оптика. Его открытия в области теории матриц изменили способ работы математиков за последние два столетия.

Мы впервые столкнулись с методом исключения Гаусса в системах линейных уравнений: две переменные. В этом разделе мы еще раз вернемся к этой технике решения систем, на этот раз с использованием матриц.

Написание расширенной матрицы системы уравнений

Матрица может служить средством представления и решения системы уравнений.Чтобы выразить систему в матричной форме, мы извлекаем коэффициенты переменных и констант, и они становятся элементами матрицы. Мы используем вертикальную линию, чтобы отделить записи коэффициентов от констант, по сути заменяя знаки равенства. Когда система написана в такой форме, мы называем ее расширенной матрицей.

Например, рассмотрим следующую систему уравнений.

Мы можем записать эту систему в виде расширенной матрицы:

Мы также можем написать матрицу, содержащую только коэффициенты.Это называется матрицей коэффициентов.

Система уравнений три на три, например

имеет матрицу коэффициентов

и представлена ​​расширенной матрицей

Обратите внимание, что матрица написана так, что переменные выстраиваются в свои собственные столбцы: термины x идут в первый столбец, y -термы во втором столбце и z -термы в третьем столбце. Очень важно, чтобы каждое уравнение было написано в стандартной форме, чтобы переменные совпадали.Если в уравнении отсутствует член переменной, коэффициент равен 0.

Для данной системы уравнений напишите расширенную матрицу.

  1. Запишите коэффициенты членов x как числа в первом столбце.
  2. Запишите коэффициенты членов y в виде чисел во втором столбце.
  3. Если есть z -термин, запишите коэффициенты как числа в третьем столбце.
  4. Нарисуйте вертикальную линию и напишите константы справа от нее.

Написание расширенной матрицы для системы уравнений

Напишите расширенную матрицу для данной системы уравнений.

Расширенная матрица отображает коэффициенты переменных и дополнительный столбец для констант.

Запишите расширенную матрицу данной системы уравнений.

Написание системы уравнений из расширенной матрицы

Мы можем использовать расширенные матрицы, чтобы помочь нам решать системы уравнений, потому что они упрощают операции, когда системы не обременены переменными. Однако важно понимать, как перемещаться между форматами, чтобы сделать поиск решений более плавным и интуитивно понятным. Здесь мы будем использовать информацию в расширенной матрице, чтобы записать систему уравнений в стандартной форме.

Напишите систему уравнений из расширенной матрицы.

Выполнение операций со строками в матрице

Теперь, когда мы можем писать системы уравнений в форме расширенной матрицы, мы рассмотрим различные операции со строками, которые могут выполняться с матрицей, такие как сложение, умножение на константу и перестановка строк.

Выполнение операций со строками в матрице — это метод, который мы используем для решения системы уравнений. Чтобы решить систему уравнений, мы хотим преобразовать матрицу в форму строки-эшелона, в которой есть единицы вниз по главной диагонали от верхнего левого угла до нижнего правого угла и нули в каждой позиции ниже главной диагонали. как показано.

Мы используем операции со строками, соответствующие операциям с уравнениями, чтобы получить новую матрицу, эквивалентную строкам в более простой форме. Вот рекомендации по получению формы рядного эшелона.

  1. В любой ненулевой строке первым ненулевым числом является 1. Оно называется ведущим 1.
  2. Любые нулевые строки помещаются внизу матрицы.
  3. Любая ведущая 1 находится ниже и правее предыдущей ведущей 1.
  4. Любой столбец, в котором в начале стоит 1, имеет нули во всех остальных позициях в столбце.

Чтобы решить систему уравнений, мы можем выполнить следующие операции со строками, чтобы преобразовать матрицу коэффициентов в форму ряда строк и выполнить обратную подстановку, чтобы найти решение.

  1. Поменяйте местами ряды. (Обозначение 🙂
  2. Умножьте строку на константу. (Обозначение 🙂
  3. Добавить произведение одной строки на константу к другой строке. (Замечание:

Каждая из строковых операций соответствует операциям, которые мы уже научились решать системы уравнений с тремя переменными. С помощью этих операций есть несколько ключевых шагов, которые быстро достигнут цели по написанию матрицы в виде эшелона строк. Чтобы получить матрицу в виде эшелона строк для поиска решений, мы используем метод исключения Гаусса, который использует операции со строками для получения 1 в качестве первой записи, так что строку 1 можно использовать для преобразования оставшихся строк.

Исключение по Гауссу

Метод исключения Гаусса относится к стратегии, используемой для получения многоуровневой формы матрицы. Цель состоит в том, чтобы записать матрицу с номером 1 в качестве записи по главной диагонали и иметь все нули внизу.

Первый шаг стратегии Гаусса включает получение 1 в качестве первой записи, так что строка 1 может использоваться для изменения строк ниже.

Учитывая расширенную матрицу, выполните операции со строками, чтобы получить форму строки-эшелона.

  1. Первое уравнение должно иметь старший коэффициент 1. Поменяйте местами строки или умножьте на константу, если необходимо.
  2. Используйте операции со строками, чтобы получить нули в первом столбце под первой записью 1.
  3. Используйте операции со строками, чтобы получить 1 в строке 2, столбце 2.
  4. Используйте операции со строками, чтобы получить нули в нижнем столбце 2, под записью 1.
  5. Используйте операции со строками, чтобы получить 1 в строке 3, столбце 3.
  6. Продолжайте этот процесс для всех строк, пока в каждой записи по главной диагонали не будет 1, а внизу будут только нули.
  7. Если какие-либо строки содержат все нули, поместите их внизу.
Решение системы методом исключения Гаусса

Решите данную систему методом исключения Гаусса.

Решите данную систему методом исключения Гаусса.

Использование исключения Гаусса для решения системы уравнений

Используйте метод исключения Гаусса для решения данной системы уравнений.

Решение зависимой системы

Решите систему уравнений.

Выполнение операций со строками в расширенной матрице 3 × 3 для получения формы Row-Echelon

Выполняет строковые операции с заданной матрицей для получения формы «строка-эшелон».

Запишите систему уравнений в виде строк.

Решение системы линейных уравнений с использованием матриц

Мы видели, как написать систему уравнений с расширенной матрицей, а затем как использовать строковые операции и обратную подстановку для получения строчно-эшелонированной формы.Теперь мы перейдем на шаг дальше от строковой формы, чтобы решить систему линейных уравнений 3 на 3. Общая идея состоит в том, чтобы исключить все переменные, кроме одной, с помощью операций со строками, а затем выполнить обратную замену для поиска других переменных.

Решение системы линейных уравнений с использованием матриц

Решите систему линейных уравнений с помощью матриц.

Решение зависимой системы линейных уравнений с использованием матриц

Решите следующую систему линейных уравнений, используя матрицы.

Решите систему, используя матрицы.

Можно ли решить любую систему линейных уравнений методом исключения Гаусса?

Да, система линейных уравнений любого размера может быть решена методом исключения Гаусса.

Для данной системы уравнений решите с помощью матриц с помощью калькулятора.

  1. Сохранить расширенную матрицу как матричную переменную
  2. Используйте функцию ref ( в калькуляторе, вызывая каждую матричную переменную по мере необходимости.

Решение систем уравнений с матрицами с помощью калькулятора

Решите систему уравнений.

Применение матриц 2 × 2 к финансам

Кэролайн инвестирует в общей сложности 12 000 фунтов стерлингов в две муниципальные облигации, одна из которых выплачивает 10,5% годовых, а другая — 12%. Годовой процент, полученный по двум инвестициям в прошлом году, составил 1335 фунтов стерлингов. Сколько было вложено по каждой ставке?

Применение матриц 3 × 3 к финансам

Ava инвестирует в общей сложности 10 000 фунтов стерлингов в три счета, один из которых платит 5% годовых, другой — 8%, а третий — 9%.Годовой процент, полученный по трем инвестициям в прошлом году, составил 770 фунтов стерлингов. Сумма, вложенная под 9%, была вдвое больше, чем сумма, вложенная под 5%. Сколько было вложено по каждой ставке?

У нас есть система трех уравнений с тремя переменными. Пусть будет сумма, вложенная под 5%, пусть будет сумма, вложенная под 8%, пусть будет сумма, вложенная под 9%. Таким образом,

В качестве матрицы имеем

Теперь мы выполняем исключение Гаусса, чтобы получить форму строки-эшелон.

В третьей строке указано usthus

.

Вторая строка говорит нам, что подставляя мы получаем

Первая строка говорит нам, что подставляем и получаем

Ответ: 3000 евро вложены под 5%, 1000 евро вложены под 8% и 6000 евро вложены под 9%.

Небольшая обувная компания взяла ссуду в размере 1 500 000 фунтов стерлингов для расширения своих запасов. Часть денег была взята под 7%, часть — под 8%, часть — под 10%. Сумма займа под 10% в четыре раза превышала сумму займа под 7%, а годовая процентная ставка по всем трем займам составляла 130 500 фунтов стерлингов. Используйте матрицы, чтобы найти сумму, заимствованную по каждой ставке.

? 150 000 при 7%, 750 000 фунтов стерлингов при 8%, 600 000 фунтов стерлингов при 10%

Ключевые концепции

  • Расширенная матрица — это матрица, которая содержит коэффициенты и константы системы уравнений.См. (Рисунок).
  • Матрица, дополненная постоянным столбцом, может быть представлена ​​как исходная система уравнений. См. (Рисунок).
  • Операции со строками включают в себя умножение строки на константу, добавление одной строки к другой строке и замену строк местами.
  • Мы можем использовать метод исключения Гаусса для решения системы уравнений. См. (Рисунок), (Рисунок) и (Рисунок).
  • Операции со строками выполняются над матрицами для получения формы «строка-эшелон». См. (Рисунок).
  • Чтобы решить систему уравнений, запишите ее в форме расширенной матрицы.Выполните операции со строками, чтобы получить форму эшелона строк. Обратный заменитель, чтобы найти решения. См. (Рисунок) и (Рисунок).
  • Калькулятор можно использовать для решения систем уравнений с использованием матриц. См. (Рисунок).
  • Многие реальные проблемы можно решить с помощью расширенных матриц. См. (Рисунок) и (Рисунок).

Упражнения по разделам

Словесный

Можно ли записать любую систему линейных уравнений в виде расширенной матрицы? Объясните, почему да или почему нет. Объясните, как написать эту расширенную матрицу.

Да. Для каждой строки коэффициенты переменных записываются поперек соответствующей строки и помещается вертикальная черта; затем константы помещаются справа от вертикальной полосы.

Можно ли записать любую матрицу в виде системы линейных уравнений? Объясните, почему да или почему нет. Объясните, как написать эту систему уравнений.

Есть только один правильный метод использования операций со строками в матрице? Попытайтесь объяснить две различные операции со строками, которые можно выполнить для расширенной матрицы

.

Нет, существует множество правильных методов использования строковых операций над матрицей.Двумя возможными способами являются следующие: (1) Поменяйте местами строки 1 и 2. Затем (2) Затем разделите строку 1 на 9.

Можно ли решить матрицу с нулевым элементом на диагонали? Объясните, почему да или почему нет. Что бы вы сделали, чтобы исправить ситуацию?

Может ли матрица с 0 элементами для всей строки иметь одно решение? Объясните, почему да или почему нет.

Нет. Матрица с 0 элементами для всей строки будет иметь либо ноль, либо бесконечно много решений.

Алгебраические

Для следующих упражнений напишите расширенную матрицу для линейной системы.

Для следующих упражнений напишите линейную систему из расширенной матрицы.

Для следующих упражнений решите систему методом исключения Гаусса.

Расширения

Для следующих упражнений используйте метод исключения Гаусса для решения системы.

Реальные приложения

Для следующих упражнений настройте расширенную матрицу, описывающую ситуацию, и найдите желаемое решение.

Ежедневно в магазине кексов продается 5 000 кексов со вкусом шоколада и ванили. Если шоколадный ароматизатор в 3 раза популярнее ванильного аромата, сколько кексов продается в день?

В конкурирующем магазине кексов ежедневно продаются кексы на сумму 4520 фунтов стерлингов.Шоколадные кексы стоят 2,25 евро, а кексы из красного бархата — 1,75 евро. Если общее количество кексов, проданных в день, составляет 2200, сколько штук каждого вкуса продается каждый день?

860 красный бархат, 1340 шоколад

Вы вложили 10 000 евро в два счета: один с простой процентной ставкой 3%, а другой — с процентной ставкой 2,5%. Если ваша общая сумма процентов по истечении одного года составила 283,50 фунтов стерлингов, какая сумма была на каждом счете по истечении года?

Вы вложили 2300 евро на счет 1 и 2700 евро на счет 2. Если общая сумма процентов по истечении одного года составляет 254 евро, а на счете 2 процентная ставка в 1,5 раза выше, чем на счете 1, каковы процентные ставки? Предположим простые процентные ставки.

4% на счет 1, 6% на счет 2

Bikes’R’Us производит велосипеды по 250 фунтов стерлингов. Производителю это обходится в 180 фунтов стерлингов за велосипед плюс стартовый взнос в размере 3500 фунтов стерлингов. Через сколько проданных велосипедов производитель выйдет на уровень безубыточности?

Крупный магазин бытовой техники рассматривает возможность приобретения пылесосов у небольшого производителя.Магазин сможет приобрести пылесосы по 86 фунтов стерлингов каждый, с оплатой доставки в размере 9 200 фунтов стерлингов, независимо от того, сколько пылесосов продано. Если магазин должен начать получать прибыль после продажи 230 единиц, сколько они должны взимать плату за пылесосы?

Три самых популярных вкуса мороженого — это шоколад, клубника и ваниль, составляющие 83% вкусов, продаваемых в магазине мороженого. Если ваниль продается на 1% больше, чем в два раза больше клубники, а шоколад продается на 11% больше, чем ваниль, сколько в общем потреблении мороженого приходится на ванильный, шоколадный и клубничный вкусы?

В магазине мороженого возрастает спрос на три вкуса.В прошлом году банановое, тыквенное и мороженое с каменистой дорогой составили 12% от общего объема продаж мороженого. В этом году на те же три вида мороженого пришлось 16,9% продаж мороженого. Продажи по каменистой дороге увеличились вдвое, продажи бананов увеличились на 50%, а продажи тыквы увеличились на 20%. Если у мороженого по каменистой дороге было на один процент меньше продаж, чем у бананового мороженого, узнайте, какой процент продаж мороженого было произведено каждым отдельным мороженым в прошлом году.

Банан составлял 3%, тыква — 7%, а каменистая дорога — 2%

Пакет с ореховой смесью содержит кешью, фисташки и миндаль.Всего в сумке 1000 орехов, а миндаля на 100 меньше, чем фисташек. Кешью весит 3 г, фисташки — 4 г, миндаль — 5 г. Если мешок весит 3,7 кг, узнайте, сколько орехов каждого вида в нем.

Пакет с ореховой смесью содержит кешью, фисташки и миндаль. Изначально в сумке было 900 орехов. Было съедено 30% миндаля, 20% кешью и 10% фисташек, и теперь в сумке осталось 770 орехов. Изначально кешью было на 100 штук больше, чем миндаля.Для начала выясните, сколько орехов каждого типа было в пакете.

100 миндальных орехов, 200 кешью, 600 фисташек

Глоссарий

дополненная матрица
матрица коэффициентов, примыкающая к столбцу констант, разделенному вертикальной линией в скобках матрицы
матрица коэффициентов
матрица, содержащая только коэффициенты из системы уравнений
Исключение по Гауссу
с использованием элементарных операций со строками для получения матрицы в виде эшелона строки
главная диагональ
записей из левого верхнего угла по диагонали в правый нижний угол квадратной матрицы
рядная форма
после выполнения строковых операций матричная форма, содержащая единицы по главной диагонали и нули в каждом пробеле ниже диагонали
эквивалент ряда
две матрицы и эквивалентны строкам, если одна может быть получена из другой путем выполнения основных операций со строками
строковые операции
: добавление одной строки к другой строке, умножение строки на константу, перестановка строк и т. Д. С целью получения формы «строка-эшелон»
Матрицы

и исключение Гаусса

Назад Замена

Напомним, что линейная система уравнений состоит из двух или более линейных уравнений с одинаковыми переменными.Линейная система, состоящая из трех уравнений стандартной формы, расположенных таким образом, что переменная x не появляется ни в одном уравнении после первого, а переменная y не появляется ни в одном уравнении после второго, называется верхнетреугольной формой. линейная система, состоящая из уравнений с тремя переменными в стандартной форме, расположенная так, что переменная x не появляется после первого уравнения, а переменная y не появляется после второго уравнения.. Например,

Обратите внимание, что система образует треугольник, в котором каждое последующее уравнение содержит на одну переменную меньше. В целом

Линейные системы в верхней треугольной форме {a1x + b1y = c1b2y = c2 {a1x + b1y + c1z = d1b2y + c2z = d2c3z = d3

Если линейная система находится в этой форме, мы можем легко найти одну из переменных, а затем произвести обратную замену, чтобы найти оставшиеся переменные.

Пример 1

Решить: {3x − y = 72y = −2.

Решение:

Напомним, что решения линейных систем с двумя переменными, если они существуют, представляют собой упорядоченные пары ( x , y ). Мы можем легко определить значение y , используя второе уравнение.

2у = −2у = −1

Затем используйте первое уравнение 3x − y = 7 и тот факт, что y = −1, чтобы найти x .

3x − y = 73x — (- 1) = 73x + 1 = 73x = 6x = 2

Ответ: (2, −1)

Пример 2

Решите: {x − 6y + 2z = 163y − 9z = 5z = −1.

Решение:

Напомним, что решения линейных систем с тремя переменными, если они существуют, являются упорядоченными тройками ( x , y , z ). Воспользуйтесь вторым уравнением 3y − 9z = 5 и тем фактом, что z = −1, чтобы найти y .

3y − 9z = 53y − 9 (−1) = 53y + 9 = 53y = −4y = −43

Затем подставьте y и z в первое уравнение.

x − 6y + 2z = 16x − 6 (−43) +2 (−1) = 16x + 8−2 = 16x + 6 = 16x = 10

Ответ: (10, −43, −1)

Попробуй! Решите: {4x − y + 3z = 12y − 9z = −23z = 2.

Ответ: (14, 2, 23)

Матрицы и исключение Гаусса

В этом разделе цель — разработать метод, упрощающий процесс решения линейных систем. Мы начинаем с определения матрицы — прямоугольного массива чисел, состоящего из строк и столбцов., Который представляет собой прямоугольный массив чисел, состоящий из строк и столбцов. Для данной линейной системы в стандартной форме мы создаем матрицу коэффициентов Матрицу коэффициентов линейной системы в стандартной форме, записанную так, как они выглядят выстроенной, без переменных или операций. записывая коэффициенты в том виде, в каком они кажутся выстроенными, без переменных или операций, как показано ниже.

Матрица коэффициентов линейной системы {a1x + b1y + c1z = d1a2x + b2y + c2z = d2a3x + b3y + c3z = d3 ⇒ [a1b1c1a2b2c2a3b3c3]

Строки представляют коэффициенты в уравнениях, а столбцы представляют коэффициенты каждой переменной. Кроме того, если мы включим столбец, представляющий константы, мы получим так называемую расширенную матрицу — матрицу коэффициентов с включенным столбцом констант.. Для линейной системы с двумя переменными

Расширенная матрица линейной системы {a1x + b1y = c1a2x + b2y = c2 ⇔ [a1b1 | c1a2b2 | c2]

А для линейной системы с тремя переменными имеем

Расширенная матрица линейной системы {a1x + b1y + c1z = d1a2x + b2y + c2z = d2a3x + b3y + c3z = d3 ⇔ [a1b1c1 | d1a2b2c2 | d2a3b3c3 | d3]

Примечание : Пунктирная вертикальная линия обеспечивает визуальное разделение между матрицей коэффициентов и столбцом констант. В других ресурсах по алгебре, с которыми вы можете столкнуться, это иногда опускается.

Пример 3

Постройте расширенную матрицу, которая соответствует: {9x − 6y = 0 − x + 2y = 1.

Решение:

Эта система состоит из двух линейных уравнений стандартной формы; следовательно, коэффициенты в матрице отображаются так же, как и в системе.

{9x − 6y = 0 − x + 2y = 1 ⇔ [9−6 | 0−12 | 1]

Пример 4

Постройте расширенную матрицу, которая соответствует: {x + 2y − 4z = 52x + y − 6z = 84x − y − 12z = 13.

Решение:

Поскольку уравнения даны в стандартной форме, коэффициенты появляются в матрице так же, как и в системе.

{x + 2y − 4z = 52x + y − 6z = 84x − y − 12z = 13 ⇔ [12−4 | 521−6 | 84−1−12 | 13]

Матрица имеет верхнюю треугольную форму, если все элементы ниже ведущего ненулевого элемента в каждой последующей строке равны нулю. Например,

Обратите внимание, что элементы ниже главной диагонали равны нулю, а коэффициенты выше образуют треугольную форму.В целом

Верхняя треугольная форма [a1b10b2] [a1b1c10b2c200c3]

Это важно, потому что в этом разделе мы очерчиваем процесс, с помощью которого можно выполнить определенные операции для создания эквивалентной линейной системы в верхней треугольной форме, чтобы ее можно было решить с помощью обратной подстановки. Обзор процесса представлен ниже:

Когда система принимает форму верхнего треугольника, мы можем использовать обратную замену, чтобы легко ее решить.Важно отметить, что представленные здесь расширенные матрицы представляют собой линейные системы уравнений в стандартной форме.

Следующие элементарные операции со строками Операции, которые могут быть выполнены для получения эквивалентных линейных систем. приводят к расширенным матрицам, которые представляют эквивалентные линейные системы:

  1. Любые две строки можно поменять местами.
  2. Каждый элемент в строке можно умножить на ненулевую константу.
  3. Любая строка может быть заменена суммой этой строки и кратной другой.

Примечание: Эти операции соответствуют свойствам, используемым в методе исключения.

Чтобы эффективно решить систему линейных уравнений, сначала постройте расширенную матрицу. Затем примените соответствующие элементарные операции со строками, чтобы получить расширенную матрицу в форме верхнего треугольника. В этой форме эквивалентная линейная система может быть легко решена с помощью обратной подстановки. Этот процесс называется гауссовским устранением. Шаги, используемые для получения эквивалентной линейной системы в верхней треугольной форме, чтобы ее можно было решить с помощью обратной подстановки., названный в честь Карла Фридриха Гаусса (1777–1855).

Рисунок 3.1

Карл Фридрих Гаусс (Википедия)

Шаги для решения линейного уравнения с двумя переменными с использованием исключения Гаусса перечислены в следующем примере.

Пример 5

Решить, используя матрицы и метод исключения Гаусса: {9x − 6y = 0 − x + 2y = 1.

Решение:

Перед началом этого процесса убедитесь, что уравнения в системе имеют стандартную форму.

Шаг 1 : Постройте соответствующую расширенную матрицу.

{9x − 6y = 0 − x + 2y = 1 ⇔ [9−6 | 0−12 | 1]

Шаг 2 : Примените операции элементарной строки, чтобы получить верхнюю треугольную форму. В этом случае нам нужно только удалить первый элемент второй строки, −1. Для этого умножьте вторую строку на 9 и прибавьте ее к первой строке.

Теперь используйте это, чтобы заменить вторую строку.

[9−6 | 0012 | 9]

В результате получается расширенная матрица в форме верхнего треугольника.

Шаг 3 : Преобразуйте обратно к линейной системе и решите, используя обратную подстановку. В этом примере у нас

[9−6 | 0012 | 9] ⇒ {9x − 6y = 012y = 9

Решите второе уравнение относительно y ,

12y = 9y = 912y = 34

Подставьте это значение вместо y в первое уравнение, чтобы найти x ,

9x − 6y = 09x − 6 (34) = 09x − 92 = 09x = 92x = 12

Ответ: (12, 34)

Шаги по использованию исключения Гаусса для решения линейного уравнения с тремя переменными перечислены в следующем примере.

Пример 6

Решить, используя матрицы и метод исключения Гаусса: {x + 2y − 4z = 52x + y − 6z = 84x − y − 12z = 13.

Решение:

Перед началом этого процесса убедитесь, что уравнения в системе имеют стандартную форму.

Шаг 1 : Постройте соответствующую расширенную матрицу.

{x + 2y − 4z = 52x + y − 6z = 84x − y − 12z = 13 ⇒ [12−4 | 521−6 | 84−1−12 | 13]

Шаг 2 : Примените операции элементарной строки, чтобы получить верхнюю треугольную форму. Начнем с исключения первого элемента второй строки, в данном случае 2. Для этого умножьте первую строку на −2, а затем прибавьте ее ко второй строке.

[12−4 | 521−6 | 84−1−12 | 13] ⇒ × (−2) −2−48−10 + 21−680−32−2

Используйте это, чтобы заменить вторую строку.

[12−4 | 50−32 | −24−1−12 | 13]

Затем удалите первый элемент третьей строки, в данном случае 4, умножив первую строку на −4 и прибавив ее к третьей строке.

[12−4 | 50−32 | −24−1−12 | 13] ⇒ × (−4) −4−816−20 + 4−1−12130−94−7

Используйте это, чтобы заменить третью строку.

[12−4 | 50−32 | −20−94 | −7]

Это приводит к расширенной матрице, в которой элементы ниже первого элемента первой строки равны нулю. Затем удалите второй элемент в третьей строке, в данном случае –9. Умножьте вторую строку на −3 и прибавьте ее к третьей строке.

Используйте это, чтобы заменить третью строку, и мы видим, что мы получили матрицу в форме верхнего треугольника.

[12−4 | 50−32 | −200−2 | −1]

Шаг 3 : Преобразуйте обратно к линейной системе и решите, используя обратную подстановку. В этом примере у нас

[12−4 | 50−32 | −200−2 | −1] ⇒ {x + 2y − 4z = 5−3y + 2z = −2−2z = −1

Ответ: Читателю остается убедиться, что решение (5,1,12).

Примечание: Обычно работа по замене строки путем умножения и сложения выполняется сбоку с использованием бумаги для заметок.

Пример 7

Решить, используя матрицы и метод исключения Гаусса: {2x − 9y + 3z = −18x − 2y − 3z = −8−4x + 23y + 12z = 47.

Решение:

Начнем с преобразования системы в расширенную матрицу коэффициентов.

{2x − 9y + 3z = −18x − 2y − 3z = −8−4x + 23y + 12z = 47 ⇒ [2−93 | −181−2−3 | −8−42312 | 47]

Элементарные операции со строками упрощаются, если ведущий ненулевой элемент в строке равен 1. По этой причине начните с того, что поменяйте местами первый и второй ряды.

Заменить строку два суммой −2, умноженной на первую и вторую строку.

Заменить третью строку суммой четырех строк первой и третьей.

Далее разделите 3-ю строку на 15.

Поменяйте местами третий ряд со вторым.

Затем замените строку 3 суммой, умноженной на 5 строк второй и третьей.

В результате получается матрица в форме верхнего треугольника. Матрица находится в форме эшелона строк Матрица в треугольной форме, где ведущий ненулевой элемент каждой строки равен 1. если она находится в верхней треугольной форме, где ведущий ненулевой элемент каждой строки равен 1. Мы можем получить эту форму, заменив третью строку на результат деления на 9.

Преобразуйте в систему линейных уравнений и решите обратной подстановкой.

[1−2−3 | −8010 | 1001 | 13] ⇒ {x − 2y − 3z = −8y = 1z = 13

Здесь y = 1 и z = 13. Подставляем в первое уравнение, чтобы найти x .

x − 2y − 3y = −8x − 2 (1) −3 (13) = — 8x − 2−1 = −8x − 3 = −8x = −5

Ответ: Следовательно, решение — (−5, 1, 13).

Технологическое примечание : Многие современные калькуляторы и системы компьютерной алгебры могут выполнять метод исключения Гаусса. Сначала вам нужно узнать, как войти в матрицу.Затем используйте функции калькулятора, чтобы найти форму эшелона строки. Предлагаем вам провести исследование по этой теме для вашей конкретной модели калькулятора.

Попробуй! Решить, используя исключение Гаусса: {x − 3y + 2z = 164x − 11y − z = 692x − 5y − 4z = 36.

Ответ: (6, −4, −1)

Напомним, что некоторые непротиворечивые линейные системы зависимы, то есть у них бесконечно много решений.А некоторые линейные системы не имеют одновременного решения; это несовместимые системы.

Пример 8

Решить, используя матрицы и метод исключения Гаусса: {x − 2y + z = 42x − 3y + 4z = 74x − 7y + 6z = 15.

Решение:

Начнем с преобразования системы в расширенную матрицу коэффициентов.

{x − 2y + z = 42x − 3y + 4z = 74x − 7y + 6z = 15 ⇒ [1−21 | 42−34 | 74−76 | 15]

Заменить вторую строку на −2 (строка 1) + (строка 2) и заменить строку три на −4 (строка 1) + (строка 3).

[1−21 | 4012 | −1012 | −1]

Заменить третью строку на −1 (строка 2) + (строка 3).

[1-21 | 4012 | -1000 | 0]

Последняя строка указывает, что это зависимая система, потому что преобразование расширенной матрицы обратно в уравнения, которые у нас есть,

{x − 2y + z = 4y + 2z = −10x + 0y + 0z = 0

Обратите внимание, что ряд нулей соответствует следующему тождеству,

0x + 0y + 0z = 00 = 0 ✓

В этом случае мы можем выразить бесконечное множество решений через z . Из второго ряда имеем:

y + 2z = −1y = −2z − 1

И из первого уравнения,

x − 2y + z = 4x − 2 (−2z − 1) + z = 4x + 5z + 2 = 4x = −5z + 2

Решения имеют вид (x, y, z) = (- 5z + 2, −2z − 1, z), где z — любое действительное число.

Ответ: (−5z + 2, −2z − 1, z)

Зависимые и несовместимые системы могут быть идентифицированы в расширенной матрице коэффициентов, когда все коэффициенты в одной строке равны нулю.

Если строка нулей имеет соответствующую константу, равную нулю, тогда матрица представляет зависимую систему. Если константа отлична от нуля, матрица представляет собой несовместимую систему.

Попробуй! Решить, используя матрицы и метод исключения Гаусса: {5x − 2y + z = −310x − y + 3z = 0−15x + 9y − 2z = 17.

Ответ: Ø

Ключевые выводы

  • Линейная система в верхней треугольной форме может быть легко решена с помощью обратной подстановки.
  • Расширенная матрица коэффициентов и метод исключения Гаусса могут использоваться для упрощения процесса решения линейных систем.
  • Чтобы решить систему с использованием матриц и исключения Гаусса, сначала используйте коэффициенты для создания расширенной матрицы. Примените операции с элементарными строками как средство для получения матрицы в форме верхнего треугольника. Преобразуйте матрицу обратно в эквивалентную линейную систему и решите ее, используя обратную подстановку.

Тематические упражнения

    Часть A: Назад Замена

      Решите, используя обратную замену.

    1. {5x − 3y = 2y = −1

    2. {3x + 2y = 1y = 3

    3. {x − 4y = 12y = −3

    4. {x − 5y = 310y = −6

    5. {4x − 3y = −167y = 0

    6. {3x − 5y = −104y = 8

    7. {2x + 3y = −13y = 2

    8. {6x − y = −34y = 3

    9. {х-у = 02у = 0

    10. {2x + y = 23y = 0

    11. {x + 3y − 4z = 1y − 3z = −2z = 3

    12. {x − 5y + 4z = −1y − 7z = 10z = −2

    13. {x − 6y + 8z = 23y − 4z = −42z = −1

    14. {2x − y + 3z = −92y + 6z = −23z = 2

    15. {10x − 3y + z = 1311y − 3z = 92z = −6

    16. {3x − 2y + 5z = −244y + 5z = 34z = −12

    17. {x − y + 2z = 12y + z = 13z = −1

    18. {x + 2y − z = 2y − 3z = 16z = 1

    19. {x − 9y + 5z = −32y = 103z = 27

    20. {4x — z = 33y − 2z = −12z = −8

    Часть B: Матрицы и исключение Гаусса

      Построить соответствующую расширенную матрицу (не решать).

    1. {х + 2у = 34х + 5у = ​​6

    2. {6x + 5y = 43x + 2y = 1

    3. {x − 2y = 12x − y = 1

    4. {х-у = 2-х + у = -1

    5. {−x + 8y = 32y = 2

    6. {3x − 2y = 4 − y = 5

    7. {3x − 2y + 7z = 84x − 5y − 10z = 6 − x − 3y + 2z = −1

    8. {x − y − z = 02x − y + 3z = −1 − x + 4y − 3z = −2

    9. {x − 9y + 5z = −32y = 103z = 27

    10. {4x − z = 33y − 2z = −12z = −8

    11. {8x + 2y = −13−2y + z = 112x − 5z = −18

    12. {x − 3z = 2y + 6z = 42x + 3y = 12

      Решите, используя матрицы и метод исключения Гаусса.

    1. {x − 5y = 22x − y = 1

    2. {x − 2y = −1x + y = 1

    3. {10x − 7y = 15−2x + 3y = −3

    4. {9x − 10y = 23x + 5y = −1

    5. {3x + 5y = 82x − 3y = 18

    6. {5x − 3y = −147x + 2y = −1

    7. {9x + 15y = 53x + 5y = 7

    8. {6x − 8y = 1−3x + 4y = −1

    9. {х + у = 0х-у = 0

    10. {7x − 3y = 03x − 7y = 0

    11. {2x − 3y = 4−10x + 15y = −20

    12. {6x − 10y = 20−3x + 5y = −10

    13. {x + y − 2z = −1 − x + 2y − z = 1x − y + z = 2

    14. {x − y + z = −2x + 2y − z = 6 − x + y − 2z = 3

    15. {2x − y + z = 2x − y + z = 2−2x + 2y − z = −1

    16. {3x − y + 2z = 7 − x + 2y + z = 6x + 3y − 2z = 1

    17. {x − 3y + z = 6 − x − y + 2z = 42x + y + z = 3

    18. {4x − y + 2z = 12x − 3y + 2z = 7−2x + 3y + 4z = −16

    19. {2x − 4y + 6z = −43x − 2y + 5z = −25x − y + 2z = 1

    20. {3x + 6y + 9z = 62x − 2y + 3z = 0−3x + 18y − 12z = 5

    21. {−x + y − z = −23x − 2y + 5z = 13x − 5y − z = 3

    22. {x + 2y + 3z = 43x + 8y + 13z = 212x + 5y + 8z = 16

    23. {2x − 4y − 5z = 3 − x + y + z = 13x − 4y − 5z = −4

    24. {5x − 3y − 2z = 43x − 6y + 4z = −6 − x + 2y − z = 2

    25. {−2x − 3y + 12z = 44x − 5y − 10z = −1 − x − 3y + 2z = 0

    26. {3x − 2y + 5z = 104x + 3y − 3z = −6x + y + z = 2

    27. {x + 2y + z = −3x + 6y + 3z = 7x + 4y + 2z = 2

    28. {2x − y + z = 14x − y + 3z = 52x + y + 3z = 7

    29. {2x + 3y − 4z = 03x − 5y + 3z = −105x − 2y + 5z = −4

    30. {3x − 2y + 9z = 2−2x − 5y − 4z = 35x − 3y + 3z = 15

    31. {8x + 2y = −13−2y + z = 112x − 5z = −18

    32. {x − 3z = 2y + 6z = 42x + 3y = 12

    33. {9x + 3y − 11z = 62x + y − 3z = 17x + 2y − 8z = 3

    34. {3x − y − z = 4−5x + y + 2z = −36x − 2y − 2z = 8

    35. {2x − 4y + 3z = 153x − 5y + 2z = 185x + 2y − 6z = 0

    36. {3x − 4y − 3z = −144x + 2y + 5z = 12−5x + 8y − 4z = −3

    Часть C: Обсуждение

    1. Изучите и обсудите историю метода исключения Гаусса. Кто первым разработал этот процесс? Опубликуйте что-нибудь, что вам показалось интересным в связи с этой историей.

    2. Изучите и обсудите историю современной матричной записи. Кому засчитывается разработка? В каких сферах они используются сегодня? Разместите свои выводы на доске обсуждений.

ответы

  1. (-15, -1)

  2. (-5, -32)

  3. (-32,23)

  4. (−6, −2, −12)

  5. (85,0, −3)

  6. (73,23, −13)

  1. [12 | 345 | 6]

  2. [1-2 | 12-1 | 1]

  3. [−18 | 302 | 2]

  4. [3−27 | 84−5−10 | 6−1−32 | −1]

  5. [1−95 | −3020 | 10003 | 27]

  6. [820 | −130−21 | 1120−5 | −18]

  7. (13, −13)

  8. (32,0)

  9. (х, 23x − 43)

  10. (12,12, −12)

  11. (1,0,12)

  12. (−8, −12z + 52, z)

  13. (-32, -12, 0)

7.

6 Решение систем с исключением Гаусса — Колледжская алгебра

Задачи обучения

В этом разделе вы:

  • Напишите расширенную матрицу системы уравнений.
  • Напишите систему уравнений из расширенной матрицы.
  • Выполняет операции со строками в матрице.
  • Решите систему линейных уравнений с помощью матриц.

Рисунок 1 Немецкий математик Карл Фридрих Гаусс (1777–1855).

Карл Фридрих Гаусс жил в конце 18 — начале 19 века, но до сих пор считается одним из самых плодовитых математиков в истории.Его вклад в математику и физику охватывает такие области, как алгебра, теория чисел, анализ, дифференциальная геометрия, астрономия и оптика. Его открытия в области теории матриц изменили способ работы математиков за последние два столетия.

Мы впервые столкнулись с методом исключения Гаусса в системах линейных уравнений: две переменные. В этом разделе мы еще раз вернемся к этой технике решения систем, на этот раз с использованием матриц.

Запись расширенной матрицы системы уравнений

Матрица может служить средством представления и решения системы уравнений.Чтобы выразить систему в матричной форме, мы извлекаем коэффициенты переменных и констант, и они становятся элементами матрицы. Мы используем вертикальную линию, чтобы отделить записи коэффициентов от констант, по сути заменяя знаки равенства. Когда система написана в такой форме, мы называем ее расширенной матрицей.

Например, рассмотрим следующую систему уравнений 2 × 22 × 2.

3x + 4y = 74x − 2y = 53x + 4y = 74x − 2y = 5

Мы можем записать эту систему в виде расширенной матрицы:

[344−2 | 75] [344−2 | 75]

Мы также можем написать матрицу, содержащую только коэффициенты.Это называется матрицей коэффициентов.

Система уравнений три на три, например

3x − y − z = 0 x + y = 5 2x − 3z = 23x − y − z = 0 x + y = 5 2x − 3z = 2

имеет матрицу коэффициентов

[3−1−111020−3] [3−1−111020−3]

и представлен расширенной матрицей

[3−1−111020−3 | 052] [3−1−111020−3 | 052]

Обратите внимание, что матрица написана так, что переменные выстраиваются в свои собственные столбцы: члены x идут в первый столбец, y -термы во втором столбце и z -термы в третьем. столбец.Очень важно, чтобы каждое уравнение было записано в стандартной форме ax + by + cz = dax + by + cz = d, чтобы переменные совпадали. Если в уравнении отсутствует член переменной, коэффициент равен 0.

How To

Для данной системы уравнений напишите расширенную матрицу.

  1. Запишите коэффициенты членов x как числа в первом столбце.
  2. Запишите коэффициенты членов y в виде чисел во втором столбце.
  3. Если есть z -термин, запишите коэффициенты как числа в третьем столбце.
  4. Нарисуйте вертикальную линию и напишите константы справа от нее.

Пример 1

Написание расширенной матрицы для системы уравнений

Напишите расширенную матрицу для данной системы уравнений.

x + 2y − z = 32x − y + 2z = 6 x − 3y + 3z = 4 x + 2y − z = 32x − y + 2z = 6 x − 3y + 3z = 4
Решение

Расширенная матрица отображает коэффициенты переменных и дополнительный столбец для констант.

[12−12−121−33 | 364] [12−12−121−33 | 364]

Попробуй # 1

Запишите расширенную матрицу данной системы уравнений.

4x − 3y = 113x + 2y = 44x − 3y = 113x + 2y = 4

Написание системы уравнений из расширенной матрицы

Мы можем использовать расширенные матрицы, чтобы помочь нам решать системы уравнений, потому что они упрощают операции, когда системы не обременены переменными. Однако важно понимать, как перемещаться между форматами, чтобы сделать поиск решений более плавным и интуитивно понятным.Здесь мы будем использовать информацию в расширенной матрице, чтобы записать систему уравнений в стандартной форме.

Пример 2

Написание системы уравнений из расширенной матричной формы

Найдите систему уравнений из расширенной матрицы.

[1−3−52−5−4−354 | −256] [1−3−52−5−4−354 | −256]
Решение

Когда столбцы представляют переменные x, x, y, y и z, z,

[1−3−52−5−4−354 | −256] → x − 3y − 5z = −22x − 5y − 4z = 5−3x + 5y + 4z = 6 [1−3−52−5−4−354 | −256] → x − 3y − 5z = −22x − 5y − 4z = 5−3x + 5y + 4z = 6

Попробуй # 2

Напишите систему уравнений из расширенной матрицы.

[1−112−13011 | 51−9] [1−112−13011 | 51−9]

Выполнение операций со строками в матрице

Теперь, когда мы можем писать системы уравнений в форме расширенной матрицы, мы рассмотрим различные операции со строками, которые могут выполняться с матрицей, такие как сложение, умножение на константу и перестановка строк.

Выполнение операций со строками в матрице — это метод, который мы используем для решения системы уравнений. Чтобы решить систему уравнений, мы хотим преобразовать матрицу в форму строки-эшелона, в которой есть единицы вниз по главной диагонали от верхнего левого угла до нижнего правого угла и нули в каждой позиции ниже главной диагонали. как показано.

Форма строки-эшелон [1ab01d001] Форма строки-эшелон [1ab01d001]

Мы используем операции со строками, соответствующие операциям с уравнениями, чтобы получить новую матрицу, эквивалентную строкам в более простой форме. Вот рекомендации по получению формы рядного эшелона.

  1. В любой ненулевой строке первым ненулевым числом является 1. Оно называется ведущим 1.
  2. Любые нулевые строки помещаются внизу матрицы.
  3. Любая ведущая 1 находится ниже и правее предыдущей ведущей 1.
  4. Любой столбец, в котором в начале стоит 1, имеет нули во всех остальных позициях в столбце.

Чтобы решить систему уравнений, мы можем выполнить следующие операции со строками, чтобы преобразовать матрицу коэффициентов в форму ряда строк и выполнить обратную подстановку, чтобы найти решение.

  1. Поменять ряды местами. (Замечание: Ri↔RjRi↔Rj)
  2. Умножьте строку на константу. (Замечание: cRicRi)
  3. Добавить произведение одной строки на константу к другой строке. (Замечание: Ri + cRj) Ri + cRj)

Каждая из строковых операций соответствует операциям, которые мы уже научились решать системы уравнений с тремя переменными.С помощью этих операций есть несколько ключевых шагов, которые быстро достигнут цели по написанию матрицы в виде эшелона строк. Чтобы получить матрицу в виде эшелона строк для поиска решений, мы используем метод исключения Гаусса, который использует операции со строками для получения 1 в качестве первой записи, так что строку 1 можно использовать для преобразования оставшихся строк.

Исключение по Гауссу

Метод исключения Гаусса относится к стратегии, используемой для получения многоуровневой формы матрицы. Цель состоит в том, чтобы записать матрицу AA с номером 1 в качестве записи вниз по главной диагонали и иметь все нули внизу.

A = [a11a12a13a21a22a23a31a32a33] → После исключения по Гауссу A = [1b12b1301b23001] A = [a11a12a13a21a22a23a31a32a33] → После исключения по Гауссу A = [1b12b1301b23001]

первый шаг может быть использован в качестве 1-й строки по Гауссу. чтобы изменить строки ниже.

Как к

Учитывая расширенную матрицу, выполните операции со строками, чтобы получить форму строки-эшелона.

  1. Первое уравнение должно иметь старший коэффициент 1.При необходимости поменяйте местами строки или умножьте на константу.
  2. Используйте операции со строками, чтобы получить нули в первом столбце под первой записью 1.
  3. Используйте операции со строками, чтобы получить 1 в строке 2, столбце 2.
  4. Используйте операции со строками, чтобы получить нули в нижнем столбце 2, под записью 1.
  5. Используйте операции со строками, чтобы получить 1 в строке 3, столбце 3.
  6. Продолжайте этот процесс для всех строк, пока в каждой записи по главной диагонали не будет 1, а внизу будут только нули.
  7. Если какие-либо строки содержат все нули, поместите их внизу.

Пример 3

Решение системы 2 × 22 × 2 методом исключения Гаусса

Решите данную систему методом исключения Гаусса.

2x + 3y = 6 x − y = 122x + 3y = 6 x − y = 12
Решение

Во-первых, мы запишем это как расширенную матрицу.

[231−1 | 612] [231−1 | 612]

Нам нужна 1 в строке 1, столбце 1. Этого можно добиться, поменяв местами строку 1 и строку 2.

R1↔R2 → [1−123 | 126] R1↔R2 → [1−123 | 126]

Теперь у нас есть 1 как первая запись в строке 1, столбце 1.Теперь давайте получим 0 в строке 2, столбце 1. Этого можно добиться, умножив строку 1 на −2, −2 и затем добавив результат к строке 2.

−2R1 + R2 = R2 → [1−105 | 125 | ] −2R1 + R2 = R2 → [1−105 | 125]

У нас есть только один шаг, чтобы умножить строку 2 на 15,15.

15R2 = R2 → [1−101 | 121] 15R2 = R2 → [1−101 | 121]

Использовать обратную замену. Вторая строка матрицы представляет y = 1. y = 1. Подставьте обратно y = 1y = 1 в первое уравнение.

x− (1) = 12 x = 32x− (1) = 12 x = 32

Решением является точка (32,1).(32,1).

Попробуй # 3

Решите данную систему методом исключения Гаусса.

4x + 3y = 11 x − 3y = −14x + 3y = 11 x − 3y = −1

Пример 4

Использование исключения Гаусса для решения системы уравнений

Используйте метод исключения Гаусса для решения заданной 2 × 22 × 2 система уравнений.

2x + y = 14x + 2y = 6 2x + y = 14x + 2y = 6
Решение

Запишите систему как расширенную матрицу.

[2142 | 16] [2142 | 16]

Получить 1 в строке 1, столбце 1.Этого можно добиться, умножив первую строку на 12,12.

12R1 = R1 → [11242 | 126] ​​12R1 = R1 → [11242 | 126] ​​

Далее нам нужен 0 в строке 2, столбце 1. Умножим строку 1 на −4−4 и прибавим строку 1 к строке 2.

−4R1 + R2 = R2 → [11200 | 124] −4R1 + R2 = R2 → [11200 | 124]

Вторая строка представляет уравнение 0 = 4,0 = 4. Следовательно, система непоследовательна и не имеет решения.

Пример 5

Решение зависимой системы

Решите систему уравнений.

3x + 4y = 126x + 8y = 243x + 4y = 126x + 8y = 24
Решение

Выполните операции со строками в расширенной матрице, чтобы попытаться получить форму строки-эшелона.

A = [3468 | 1224] A = [3468 | 1224] −12R2 + R1 = R1 → [0068 | 024] R1↔R2 → [6800 | 240] −12R2 + R1 = R1 → [0068 | 024] R1↔ R2 → [6800 | 240]

Матрица заканчивается всеми нулями в последней строке: 0y = 0,0y = 0. Таким образом, существует бесконечное количество решений и система классифицируется как зависимая. Чтобы найти общее решение, вернитесь к одному из исходных уравнений и решите относительно y.y.

3x + 4y = 12 4y = 12−3x y = 3−34x3x + 4y = 12 4y = 12−3x y = 3−34x

Итак, решение этой системы — (x, 3−34x).(x, 3−34x).

Пример 6

Выполнение операций со строками в расширенной матрице 3 × 3 для получения формы Row-Echelon

Выполняет строковые операции с заданной матрицей для получения формы «строка-эшелон».

[1−342−56−334 | 366] [1−342−56−334 | 366]
Решение

В первой строке уже есть 1 в строке 1, столбце 1. Следующим шагом будет умножение строки 1 на −2−2 и прибавление ее к строке 2. Затем замените строку 2 результатом.

−2R1 + R2 = R2 → [1−3401−2−334 | 306] −2R1 + R2 = R2 → [1−3401−2−334 | 306]

Затем получите ноль в строке 3, столбце 1.

3R1 + R3 = R3 → [1−3401−20−616 | 3015] 3R1 + R3 = R3 → [1−3401−20−616 | 3015]

Затем получаем ноль в строке 3, столбце 2.

6R2 + R3 = R3 → [1−3401−2004 | 3015] 6R2 + R3 = R3 → [1−3401−2004 | 3015]

Последний шаг — получить 1 в строке 3, столбце 3.

14R3 = R3 → [1−3401−2001 | 3−6154] 14R3 = R3 → [1−3401−2001 | 3−6154]

Попробуй # 4

Запишите систему уравнений в виде строк.

x − 2y + 3z = 9 − x + 3y = −42x − 5y + 5z = 17 x − 2y + 3z = 9 − x + 3y = −42x − 5y + 5z = 17

Решение системы линейных уравнений с использованием матриц

Мы видели, как написать систему уравнений с расширенной матрицей, а затем как использовать строковые операции и обратную подстановку для получения строчно-эшелонированной формы.Теперь мы перейдем на шаг дальше от строковой формы, чтобы решить систему линейных уравнений 3 на 3. Общая идея состоит в том, чтобы исключить все переменные, кроме одной, с помощью операций со строками, а затем выполнить обратную замену для поиска других переменных.

Пример 7

Решение системы линейных уравнений с использованием матриц

Решите систему линейных уравнений с помощью матриц.

x − y + z = 82x + 3y − z = −23x − 2y − 9z = 9x − y + z = 82x + 3y − z = −23x − 2y − 9z = 9
Решение

Сначала мы пишем расширенную матрицу.

[1−1123−13−2−9 | 8−29] [1−1123−13−2−9 | 8−29]

Затем мы выполняем строковые операции для получения формы «строка-эшелон».

−2R1 + R2 = R2 → [1−1105−33−2−9 | 8−189] −3R1 + R3 = R3 → [1−1105−301−12 | 8−18−15] −2R1 + R2 = R2 → [1−1105−33−2−9 | 8−189] −3R1 + R3 = R3 → [1−1105−301−12 | 8−18−15]

Самый простой способ получить 1 в строке 2 столбец 1 предназначен для замены R2R2 и R3.R3.

Обмен R2 и R3 → [1−11801−12−1505−3−18] Обмен R2andR3 → [1−11801−12−1505−3−18]

Тогда

−5R2 + R3 = R3 → [1−1101−120057 | 8−1557] −157R3 = R3 → [1−1101−12001 | 8−151] −5R2 + R3 = R3 → [1−1101−120057 | 8− 1557] −157R3 = R3 → [1−1101−12001 | 8−151]

Последняя матрица представляет собой эквивалентную систему.

x − y + z = 8 y − 12z = −15 z = 1x − y + z = 8 y − 12z = −15 z = 1

Используя обратную подстановку, мы получаем решение как (4, −3,1) . (4, −3,1).

Пример 8

Решение зависимой системы линейных уравнений с использованием матриц

Решите следующую систему линейных уравнений, используя матрицы.

−x − 2y + z = −1 2x + 3y = 2y − 2z = 0 − x − 2y + z = −1 2x + 3y = 2y − 2z = 0
Решение

Запишите расширенную матрицу.

[−1−2123001−2 | −120] [- 1−2123001−2 | −120]

Сначала умножьте строку 1 на −1−1, чтобы получить 1 в строке 1, столбце 1.Затем выполните операции со строками, чтобы получить форму эшелона строк.

−R1 → [12−123001−2 | 120] −R1 → [12−123001−2 | 120] R2↔R3 → [12−101−2230 | 102] R2↔R3 → [12−101−2230 | 102] −2R1 + R3 = R3 → [12−101−20−12 | 100] −2R1 + R3 = R3 → [12−101−20−12 | 100] R2 + R3 = R3 → [12−101−2000 | 210] R2 + R3 = R3 → [12−101−2000 | 210]

Последняя матрица представляет следующая система.

x + 2y − z = 1 y − 2z = 0 0 = 0x + 2y − z = 1 y − 2z = 0 0 = 0

Мы видим из тождества 0 = 00 = 0, что это зависимая система с бесконечным числом решений. Затем мы находим общее решение. Решив второе уравнение для yy и подставив его в первое уравнение, мы можем решить для zz через x.x.

x + 2y − z = 1 y = 2zx + 2 (2z) −z = 1 x + 3z = 1 z = 1 − x3x + 2y − z = 1 y = 2zx + 2 (2z) −z = 1 x + 3z = 1 z = 1 − x3

Теперь мы подставляем выражение для zz во второе уравнение, чтобы решить относительно yy через xx

y − 2z = 0z = 1 − x3y − 2 (1 − x3) = 0y = 2−2x3y − 2z = 0z = 1 − x3y − 2 (1 − x3) = 0y = 2−2×3

Общее решение: ( x, 2−2×3,1 − x3).(х, 2−2×3,1 − x3).

Попробуй # 5

Решите систему, используя матрицы.

x + 4y − z = 42x + 5y + 8z = 15x + 3y − 3z = 1x + 4y − z = 42x + 5y + 8z = 15x + 3y − 3z = 1

Вопросы и ответы

Можно ли решить любую систему линейных уравнений методом исключения Гаусса?

Да, система линейных уравнений любого размера может быть решена методом исключения Гаусса.

Как к

Для данной системы уравнений решите с помощью матриц с помощью калькулятора.

  1. Сохранить расширенную матрицу как матричную переменную [A], [B], [C],….[A], [B], [C],….
  2. Используйте функцию ref ( в калькуляторе, вызывая каждую матричную переменную по мере необходимости.

Пример 9

Решение систем уравнений с матрицами с помощью калькулятора

Решите систему уравнений.

5x + 3y + 9z = −1−2x + 3y − z = −2 − x − 4y + 5z = 1 5x + 3y + 9z = −1−2x + 3y − z = −2 − x − 4y + 5z = 1
Решение

Напишите расширенную матрицу для системы уравнений.

[539−23−1−1−45 | −1−2−1] [539−23−1−1−45 | −1−2−1]

На странице матриц калькулятора введите расширенную матрицу выше в качестве матричной переменной [A].[А].

[A] = [539−1−23−1−2−1−451] [A] = [539−1−23−1−2−1−451]

Используйте ссылку (функция в калькуляторе, вызов матричной переменной [A]. [A].

Оценить.

[1359515011321−47001−24187] → x + 35y + 95z = −15y + 1321z = −47z = −24187 [1359515011321−47001−24187] → x + 35y + 95z = −15y + 1321z = −47z = −24187

Использование обратная подстановка, решение будет (61187, −

, −24187). (61187, −

, −24187).

Пример 10

Применение матриц 2 × 2 к финансам

Кэролайн инвестирует в общей сложности 12 000 долларов в две муниципальные облигации, одна из которых выплачивает 10.5% годовых, а другой — 12% годовых. Годовой процент, полученный по двум инвестициям в прошлом году, составил 1335 долларов. Сколько было вложено по каждой ставке?

Решение

У нас есть система двух уравнений с двумя переменными. Пусть x = x = сумма, инвестированная под 10,5%, а y = y = сумма, инвестированная под 12%.

x + y = 12,0000,105x + 0,12y = 1,335 x + y = 12,0000,105x + 0,12y = 1,335

В качестве матрицы мы имеем

[110.1050.12 | 12,0001,335] [110.1050.12 | 12,0001,335]

Умножьте строку 1 на −0,105−0,105 и прибавьте результат к строке 2.

[1100.015 | 12,00075] [1100.015 | 12,00075]

Затем,

0,015y = 75 y = 5,0000,015y = 75 y = 5,000

Итак, 12,000-5,000 = 7,000. 12,000-5,000 = 7,000.

Таким образом, 5000 долларов были инвестированы под 12% и 7000 долларов под 10,5%.

Пример 11

Применение матриц 3 × 3 к финансам

Ava инвестирует в общей сложности 10 000 долларов в три счета, один из которых платит 5% годовых, другой — 8%, а третий — 9%.Годовой процент, полученный по трем инвестициям в прошлом году, составил 770 долларов. Сумма, вложенная под 9%, была вдвое больше, чем сумма, вложенная под 5%. Сколько было вложено по каждой ставке?

Решение

У нас есть система трех уравнений с тремя переменными. Пусть xx будет суммой, инвестированной под 5%, пусть yy будет суммой, инвестированной под 8%, и пусть zz будет суммой, инвестированной под 9%. Таким образом,

x + y + z = 10,0000,05x + 0,08y + 0,09z = 770 2x − z = 0 x + y + z = 10,0000.05x + 0,08y + 0,09z = 770 2x − z = 0

В качестве матрицы имеем

[1110.050.080.0920−1 | 10,0007700] [1110. 050.080.0920−1 | 10,0007700]

Теперь мы выполняем исключение по Гауссу, чтобы получить форму строки-эшелон.

−0.05R1 + R2 = R2 → [11100.030.0420−1 | 10,0002700] −2R1 + R3 = R3 → [11100.030.040−2−3 | 10,000270−20,000] 10.03R2 = R2 → [01101430−2 −3 | 10,0009,000−20,000] 2R2 + R3 = R3 → [111014300−13 | 10,0009,000−2,000] −0,05R1 + R2 = R2 → [11100.030.0420−1 | 10,0002700] — 2R1 + R3 = R3 → [11100.030.040−2−3 | 10,000270−20,000] 10.03R2 = R2 → [01101430−2−3 | 10,0009,000−20,000] 2R2 + R3 = R3 → [111014300−13 | 10,0009,000−2,000]

Третья строка сообщает нам -13z = −2,000; −13z = −2,000; таким образом, z = 6000. z = 6000.

Вторая строка говорит нам, что y + 43z = 9000.y + 43z = 9000. Подставляя z = 6000, z = 6000, мы получаем

y + 43 (6000) = 9000y + 8000 = 9000y = 1000y + 43 (6000) = 9000y + 8000 = 9000y = 1000

. х + у + г = 10,000. х + у + г = 10,000. Подставив y = 1,000y = 1,000 и z = 6,000, z = 6,000, мы получим

x + 1,000 + 6,000 = 10,000 x = 3,000x + 1,000 + 6,000 = 10,000 x = 3,000

Ответ: 3,000 долларов вложены под 5% годовых, 1000 долларов инвестировано под 8%, а 6000 долларов — под 9%.

Попробуй # 6

Небольшая обувная компания взяла ссуду в размере 1 500 000 долларов на расширение своих запасов. Часть денег была взята под 7%, часть — под 8%, часть — под 10%. Сумма займа под 10% в четыре раза превышала сумму займа под 7%, а годовая процентная ставка по всем трем займам составляла 130 500 долларов. Используйте матрицы, чтобы найти сумму, заимствованную по каждой ставке.

7.6 Упражнения по разделам

Устные
1.

Можно ли записать любую систему линейных уравнений в виде расширенной матрицы? Объясните, почему да или почему нет.Объясните, как написать эту расширенную матрицу.

2.

Можно ли любую матрицу записать в виде системы линейных уравнений? Объясните, почему да или почему нет. Объясните, как написать эту систему уравнений.

3.

Есть только один правильный метод использования строк в матрице? Попытайтесь объяснить две различные операции со строками, которые можно использовать для решения расширенной матрицы [931−2 | 06]. [931−2 | 06].

4.

Можно ли решить матрицу с нулевым элементом на диагонали? Объясните, почему да или почему нет. Что бы вы сделали, чтобы исправить ситуацию?

5.

Может ли матрица с 0 элементами для всей строки иметь одно решение? Объясните, почему да или почему нет.

Алгебраические

Для следующих упражнений напишите расширенную матрицу для линейной системы.

6.

8x − 37y = 82x + 12y = 38x − 37y = 82x + 12y = 3

7.

16y = 49x − y = 2 16y = 49x − y = 2

8.

3x + 2y + 10z = 3−6x + 2y + 5z = 13 4x + z = 183x + 2y + 10z = 3−6x + 2y + 5z = 13 4x + z = 18

9.

x + 5y + 8z = 1912x + 3y = 43x + 4y + 9z = −7 x + 5y + 8z = 1912x + 3y = 43x + 4y + 9z = −7

10.

6x + 12y + 16z = 4 19x − 5y + 3z = −9 x + 2y = −86x + 12y + 16z = 4 19x − 5y + 3z = −9 x + 2y = −8

Для следующих упражнений напишите линейную систему из расширенной матрицы.

11.

[−256−18 | 526] [- 256−18 | 526]

12.

[341017 | 10439] [341017 | 10439]

13.

[320−1−94857 | 3−18] [320−1−94857 | 3−18]

14.

[8291−175003 | 433810] [8291−175003 | 433810]

15.

[45−2015887−3 | 122−5] [45−2015887−3 | 122-5]

Для следующих упражнений решите систему методом исключения Гаусса.

16.

[1000 | 30] [1000 | 30]

17.

[1010 | 12] [1010 | 12]

18.

[1245 | 36] [1245 | 36]

19.

[−124−5 | −36] [- 124−5 | −36]

20.

[−2002 | 1−1] [- 2002 | 1−1]

21.

2x − 3y = −95x + 4y = 58 2x − 3y = −95x + 4y = 58

22.

6x + 2y = −43x + 4y = −176x + 2y = −43x + 4y = −17

23.

2x + 3y = 12 4x + y = 142x + 3y = 12 4x + y = 14

24.

−4x − 3y = −2 3x − 5y = −13−4x − 3y = −2 3x − 5y = −13

25.

−5x + 8y = 310x + 6y = 5−5x + 8y = 310x + 6y = 5

26.

3x + 4y = 12−6x − 8y = −24 3x + 4y = 12−6x − 8y = −24

27.

−60x + 45y = 12 20x − 15y = −4−60x + 45y = 12 20x − 15y = −4

28.

11x + 10y = 4315x + 20y = 6511x + 10y = 4315x + 20y = 65

29.

2x − y = 23x + 2y = 172x − y = 23x + 2y = 17.

30.

−1.06x − 2.25y = 5.51−5.03x − 1.08y = 5.40−1.06x − 2.25y = 5.51−5.03x − 1.08y = 5,40

31.

34x − 35y = 414x + 23y = 134x − 35y = 414x + 23y = 1

32.

14x − 23y = −112x + 13y = 314x − 23y = −112x + 13y = 3

33.

[100011001 | 314587] [100011001 | 314587]

34.

[101110011 | 5020−90] [101110011 | 5020−90]

35.

[123056008 | 479] [123056008 | 479]

36.

[−0.10.3−0.1−0.40.20.10.60.10.7 | 0.20.8−0.8] [- 0.10.3−0.1−0.40.20.10.60.10.7 | 0,20,8−0,8]

37.

−2x + 3y − 2z = 3 4x + 2y − z = 94x − 8y + 2z = −6−2x + 3y − 2z = 3 4x + 2y − z = 94x − 8y + 2z = −6

38.

x + y − 4z = −4 5x − 3y − 2z = 0 2x + 6y + 7z = 30 x + y − 4z = −4 5x − 3y − 2z = 0 2x + 6y + 7z = 30

39.

2x + 3y + 2z = 1 −4x − 6y − 4z = −210x + 15y + 10z = 5 2x + 3y + 2z = 1 −4x − 6y − 4z = −210x + 15y + 10z = 5

40.

x + 2y − z = 1 − x − 2y + 2z = −23x + 6y − 3z = 5 x + 2y − z = 1 − x − 2y + 2z = −23x + 6y − 3z = 5

41.

x + 2y − z = 1 − x − 2y + 2z = −23x + 6y − 3z = 3 x + 2y − z = 1 − x − 2y + 2z = −23x + 6y − 3z = 3

42.

x + y = 2 x + z = 1 − y − z = −3x + y = 2 x + z = 1 − y − z = −3

43.

x + y + z = 100 x + 2z = 125 − y + 2z = 25x + y + z = 100 x + 2z = 125 − y + 2z = 25.

44.

14x − 23z = −1215x + 13y = 4715y − 13z = 2914x − 23z = −1215x + 13y = 4715y − 13z = 29

45.

−12x + 12y + 17z = −5314 12x − 12y + 14z = 3 14x + 15y + 13z = 2315−12x + 12y + 17z = −5314 12x − 12y + 14z = 3 14x + 15y + 13z = 2315

46.

−12x − 13y + 14z = −296 15x + 16y − 17z = 431210−18x + 19y + 110z = −4945−12x − 13y + 14z = −296 15x + 16y − 17z = 431210−18x + 19y + 110z = — 4945

Расширения

Для следующих упражнений используйте метод исключения Гаусса для решения системы.

47.

x − 17 + y − 28 + z − 34 = 0x + y + z = 6x + 23 + 2y + z − 33 = 5x − 17 + y − 28 + z − 34 = 0x + y + z = 6x. + 23 + 2у + г — 33 = 5

48.

x − 14 − y + 14 + 3z = −1 x + 52 + y + 74 − z = 4 x + y − z − 22 = 1x − 14 − y + 14 + 3z = −1 x + 52 + y + 74-г = 4 х + у-г-22 = 1

49.

x − 34 − y − 13 + 2z = −1x + 52 + y + 52 + z + 52 = 8x + y + z = 1x − 34 − y − 13 + 2z = −1x + 52 + y + 52 + z + 52 = 8х + у + г = 1

50.

x − 310 + y + 32−2z = 3x + 54 − y − 18 + z = 32x − 14 + y + 42 + 3z = 32x − 310 + y + 32−2z = 3x + 54 − y − 18 + z = 32x − 14 + y + 42 + 3z = 32

51.

x − 34 − y − 13 + 2z = −1x + 52 + y + 52 + z + 52 = 7x + y + z = 1x − 34 − y − 13 + 2z = −1x + 52 + y + 52 + z + 52 = 7x + y + z = 1

Реальные приложения

Для следующих упражнений настройте расширенную матрицу, описывающую ситуацию, и найдите желаемое решение.

52.

Ежедневно в магазине кексов продается 5 000 кексов со вкусом шоколада и ванили. Если шоколадный ароматизатор в 3 раза популярнее ванильного аромата, сколько кексов продается в день?

53.

В конкурирующем магазине кексов ежедневно продаются кексы на 4520 долларов. Шоколадные кексы стоят 2,25 доллара, а кексы из красного бархата — 1,75 доллара. Если общее количество кексов, проданных в день, составляет 2200, сколько штук каждого вкуса продается каждый день?

54.

Вы вложили 10 000 долларов в два счета: один с простой процентной ставкой 3%, а другой — с процентной ставкой 2,5%. Если ваша общая сумма процентов по истечении одного года составила 283,50 доллара, сколько денег было на каждом счете по прошествии года?

55.

Вы вложили 2300 долларов на счет 1 и 2700 долларов на счет 2.Если общая сумма процентов по истечении одного года составляет 254 доллара, а на счете 2 процентная ставка в 1,5 раза выше, чем на счете 1, каковы процентные ставки? Предположим простые процентные ставки.

56.

Bikes’R’Us производит велосипеды по 250 долларов. Он стоит производителю 180 долларов за велосипед плюс стартовый взнос в размере 3500 долларов. Через сколько проданных велосипедов производитель выйдет на уровень безубыточности?

57.

Крупный магазин бытовой техники рассматривает возможность покупки пылесосов у небольшого производителя. Магазин сможет приобрести пылесосы по цене 86 долларов за каждый, с оплатой доставки в размере 9 200 долларов, независимо от того, сколько пылесосов будет продано. Если магазин должен начать получать прибыль после продажи 230 единиц, сколько они должны взимать плату за пылесосы?

58.

Три самых популярных вкуса мороженого — это шоколад, клубника и ваниль, составляющие 83% вкусов, продаваемых в магазине мороженого. Если ваниль продается на 1% больше, чем в два раза больше клубники, а шоколад продается на 11% больше, чем ваниль, сколько в общем потреблении мороженого приходится на ванильный, шоколадный и клубничный вкусы?

59.

В магазине мороженого возрастает спрос на три вкуса.В прошлом году банановое, тыквенное и мороженое с каменистой дорогой составили 12% от общего объема продаж мороженого. В этом году на те же три вида мороженого пришлось 16,9% продаж мороженого. Продажи по каменистой дороге увеличились вдвое, продажи бананов увеличились на 50%, а продажи тыквы увеличились на 20%. Если у мороженого по каменистой дороге было на один процент меньше продаж, чем у бананового мороженого, узнайте, какой процент продаж мороженого было произведено каждым отдельным мороженым в прошлом году.

60.

Пакет с ореховой смесью содержит кешью, фисташки и миндаль.Всего в сумке 1000 орехов, а миндаля на 100 меньше, чем фисташек. Кешью весит 3 г, фисташки — 4 г, миндаль — 5 г. Если мешок весит 3,7 кг, узнайте, сколько орехов каждого вида в нем.

61.

Пакет с ореховой смесью содержит кешью, фисташки и миндаль. Изначально в сумке было 900 орехов. Было съедено 30% миндаля, 20% кешью и 10% фисташек, и теперь в сумке осталось 770 орехов. Изначально кешью было на 100 штук больше, чем миндаля.Для начала выясните, сколько орехов каждого типа было в пакете.

Метод Гаусса-Джордана — обзор

Пример 2.2.3

Инверсия матрицы Гаусса-Жордана

Метод Гаусса-Джордана основан на том факте, что существуют матрицы M L , такие что произведение M L A оставит произвольную матрицу A без изменений, за исключением

(a)

одна строка, умноженная на константу, или

(b)

одна строка заменена исходной строкой за вычетом числа, кратного другой ряд, или

(с)

чередование двух рядов.

Фактические матрицы M L , которые выполняют эти преобразования, являются предметом упражнения 2.2.21.

Используя эти преобразования, строки матрицы могут быть изменены (путем умножения матриц) теми же способами, которыми мы могли изменять элементы определителей, поэтому мы можем действовать аналогично тем, которые используются для редукции определителей по Гауссу. устранение. Если A неособое число, применение последовательности M L , т.е.е., M = (… ML′′ML′ML), может приводить A к единичной матрице:

MA = 1 или M = A − 1.

Таким образом, нам нужно применять последовательные преобразования к A до тех пор, пока эти преобразования не уменьшат A до 1 , отслеживая продукт этих преобразований. Мы следим за последовательным применением преобразований к единичной матрице.

Вот конкретный пример. Мы хотим инвертировать матрицу

A = (321231114).

Наша стратегия будет заключаться в том, чтобы записать рядом друг с другом матрицу A и единичную матрицу одинакового размера и выполнять одни и те же операции с каждой, пока A не будет преобразована в единичную матрицу, что означает, что единичная матрица будет иметь изменен на A −1 . Начнем с

(321231114) и (100010001).

Умножаем строки по мере необходимости, чтобы установить в единицу все элементы первого столбца левой матрицы:

(1231313212114) и (13000120001).

Вычитая первую строку из второй и третьей строк, получаем

(1231305616013113) и (1300−13120−1301).

Затем мы делим вторую строку (из обеих матриц ) на 56 и вычитаем ее в 23 раза из первой строки и в 13 раз из третьей строки. Результаты для обеих матриц равны

(1015011500185) и (35-250-25350-15-151).

Мы делим третью строку (из обеих матриц ) на 185. Затем на последнем шаге 15 раз третья строка вычитается из каждой из первых двух строк (обеих матриц). Наша последняя пара —

(100010001) и A − 1 = (1118−718−118−7181118−118−118−118518).

Мы можем проверить нашу работу, умножив исходное A на вычисленное A −1 , чтобы увидеть, действительно ли мы получаем единичную матрицу 1 .

ИСКЛЮЧЕНИЕ ПО ГАУССУ: РЕШЕНИЕ СИСТЕМ ЛИНЕЙНЫХ УРАВНЕНИЙ: ПРИМЕРЫ И РЕШАЕМЫЕ ЗАДАЧИ: СТАРШАЯ ШКОЛА

Содержимое этой страницы:


Введение

Система уравнений (линейная) — это группа (линейных) уравнений с различные неизвестные факторы. Вообще говоря, неизвестные факторы входят в различные уравнения.

Уравнение с различными неизвестными факторами связывает их друг с другом.

Решение системы состоит в нахождении значения неизвестных факторов способом, который проверяет — все уравнений, составляющих систему.

  • Если существует одно решение (одно значение для каждого неизвестного фактора), мы будем говорить, что система Согласованная независимая система (СНГ) .

  • Если существуют различные решения (система имеет бесконечно много решений), мы говорим, что система является Согласованная зависимая система (CDS). .

  • Если решения нет, а это произойдет, если есть два или несколько уравнений, которые нельзя проверить одновременно, мы говорим, что это несовместимая система (IS) .Например, следующая система уравнений

    $$ \ begin {case} \ begin {array} {lcl} y & = & 0 \\ 2x + y & = & 0 \\ 2x + y & = & 2 \ end {array} \ end {ases} $ $

    несовместимо, потому что мы получаем решение x = 0 из второго уравнения и, из третьего, x = 1 .

В этом разделе мы собираемся решать системы, используя метод исключения Гаусса , который заключается в простом выполнении элементарных операций в строке или столбце расширенной матрицы, чтобы получить свой эшелон из или его сокращенный эшелон форма (Гаусс-Иордан).


Метод разрешения

  1. Применяем метод исключения Гаусса-Джордана : получаем уменьшенная форма эшелона строки из расширенной матрицы систему уравнений, выполняя элементарные операции в строках (или столбцах).

  2. Получив матрицу, мы применяем теорему Руше-Капелли , чтобы определить тип системы и получить решение (я), а именно:

    Пусть A · X = B будет системой m линейных уравнений с n неизвестными факторы, m и n натуральные числа (не ноль):

    • AX = B соответствует тогда и только тогда, когда,

      $$ ранг (A) = ранг (A | B) $$

    • AX = B является непротиворечивым независимым тогда и только тогда, когда,

      $$ ранг (A) = n = ранг (A | B) $$

Примечание: Элементарные операции в строках или столбцах позволяют получить системы, эквивалентные исходной, но с формой, упрощающей получение решений (если они есть). Также есть более быстрые инструменты для выработки решений в СНГ, такие как правило Крамера.




Система 1

Показать решение

Расширенная матрица системы

того же размера, что и система (2×3). Вертикальная линия, отделяющая матричные коэффициенты от вектора независимых членов.

Выполняем элементарные операции в строках для получения приведенной формы эшелона строк:

Умножаем первую строку на 1/5 а вторую на 1/3

Добавляем вторую строку с первой

Вторую строку умножаем на 5/7

Складываем первую строку со второй, умноженной на -2/5

Эта последняя эквивалентная матрица представлена ​​в сокращенной форме эшелона строк. и это позволяет нам быстро увидеть рейтинг матрица коэффициентов и дополненная.

Считаем ранги:

По теореме Руше-Капелли система непротиворечива Независимая. Полученная матрица представляет собой систему

, который является решением исходной системы.



Система 2

Показать решение

Расширенная матрица системы

Проводим элементарные операции в строках для получения приведенных форма эшелона строки:

Вторую строку умножаем на 1/2

Первый ряд складываем со вторым

Умножаем первую строку на 1/3

Эта последняя эквивалентная матрица находится в форме сокращенного эшелона строк и имеет нулевую строку, что означает, что строки в исходной системе линейно зависимы. (любой из них может быть получен путем умножения другого на скаляр, не равный нулю).

Рассчитываем ранги

По теореме Руше-Капелли система непротиворечива. Кроме того, это зависит, потому что ранг (1) ниже, чем количество неизвестных факторов (2).

Полученная матрица представляет собой систему

Решения



Система 3

Показать решение

Расширенная матрица системы

Выполняем элементарные операции в строках для получения приведенной формы эшелона строк

Мы меняем порядок строк (так у нас уже будет 1 в первой строке без необходимости умножать)

Складываем вторую строку с первой, умноженной на 5 :

Вторую строку умножаем на -1/15

Эта последняя матрица имеет эшелонированную форму (не сокращена).

Мы можем непосредственно заметить, что система несовместима, потому что у нас следующее равенство (вторая строка):

$$ 0x + 0y = 1 $$

, что невозможно равенство.

Рассчитываем ранги матрицы коэффициентов и дополненной:

По теореме Руше-Капелли система несовместна (решения нет). Полученная матрица представляет собой систему

Система непоследовательна, потому что мы имеем невозможное равенство

$$ 0 = 1 $$



Система 4

Показать решение

Расширенная матрица системы

(размер 3х4).

Выполняем элементарные операции в строках для получения приведенной формы эшелона строк

Умножаем первую строку на 1/5

Складываем вторую и третью строки, умножив первую на -2

Вторую и третью строки умножаем на 5

Складываем вторую строку с третьей, умноженной на -1

Умножаем вторую строку на -1/10 , а третью на 1/11

Складываем первую строку со второй, умноженной на -2/5 , а третью со второй, умноженной на -1

Третью строку умножаем на -11/5

Эта последняя эквивалентная матрица находится в сокращенной форме эшелона строк (мы знаем ее, потому что это единичная матрица). Имея единичную матрицу, мы знаем, что это непротиворечивая независимая система, и можем получить единственное решение.

Рассчитываем ранги

По теореме Руше-Капелли система непротиворечива Независимая. Полученная матрица представляет собой систему

, который является решением системы.



Система 5

Показать решение

Расширенная матрица системы

Выполняем элементарные операции в строках для получения приведенной формы эшелона строк

Мы вычитаем первую строку из второй и добавляем третью строку с первой.

Умножаем первую строку на 1/2 , а вторую на 1/3

Мы складываем первую строку со второй, умноженной на 1/2 , и третью строку со второй, умноженной на -2

Третью строку умножаем на 1/3

Складываем первую строку с третьей, умноженной на -3/2

Эта последняя матрица представляет собой эшелонированную матрицу с сокращенным числом строк (мы знаем это, потому что у нас есть единичная матрица).

Рассчитываем ранги

По теореме Руше-Капелли система непротиворечива Независимая. Полученная матрица представляет собой систему



Система 6

Показать решение

Расширенная матрица системы

Выполняем элементарные операции в строках для получения приведенной формы эшелона строк

Умножаем вторую строку на -1/3 и третью на 1/4

Складываем вторую и третью строки, умножив первую на -1

Умножаем вторую строку на -3/4 , а третью на -1

Третью строку складываем со второй, умноженной на -1

Эта последняя матрица имеет эшелонированную форму (но не сокращена) и мы не продолжаем выполнять операции по строкам, потому что можем видим, что последняя строка делает систему непоследовательной. Эта строка сообщает нам:

$$ 0x + 0y + 0z = -1 $$

И все это невозможное равенство.

Фактически у нас есть ранги

По теореме Руше-Капелли система несовместна.



Система 7

Показать решение

Расширенная матрица системы

Выполняем элементарные операции в строках для получения приведенной формы эшелона строк

Умножаем первую и вторую строки на 1/3 и 1/6 соответственно

Вторую строку вычитаем из третьей

Добавляем первую строку со второй

Третью строку умножаем на 3

Эта последняя матрица находится в сокращенном эшелоне строк формы , поэтому мы можем легко вычислить ранги:

Рассчитываем ранги

По теореме Руше-Капелли система непротиворечива. Но он не является независимым, потому что количество неизвестных факторов (3) отличается от ранга. Полученная матрица представляет собой систему

Решения



Система 8

Показать решение

Расширенная матрица системы

Примечание: перед тем, как мы начнем, отметим, что процедура будет таким же, как и было до сих пор.Но у нас есть корни в матрице, а это значит, что операции над строками будут длинными и утомительно. Эта задача не очень интересна в дидактическом смысле, помимо расчетов.

Проводим элементарные операции в строках, чтобы получить пониженный рядный эшелон формы

Мы складываем вторую строку с первой, умноженной на -√5 и, с третья, умноженная на -2/5

Умножаем вторую строку на 1 / √5 , а третью на 5/17

К первой прибавляем третью и ко второй вычитаем третью

Переписываем матрицу

Вторую строку умножаем на (√5 / 5-5) -1

Складываем первую строку со второй, умноженной на -5

В этой последней матрице (почти) в строке уменьшено форма эшелона (надо поменять второй и третий ряды так что это правда). Из последней матрицы получаем решения:



Система 9

Показать решение

Расширенная матрица системы

Выполняем элементарные операции в строках для получения приведенной формы эшелона строк

Складываем третью и четвертую строки, умножая первую на 3 и на -2 соответственно

Складываем первую, вторую и третью строки с четвертой, умноженной на 2 , -3 и 5 соответственно

Четвертую строку умножаем на -1 и меняем на вторую

Умножаем третью и четвертую строки на 1/34 и -1/22 соответственно

Вычитаем третью строку из четвертой

Умножаем четвертую на -187/42

К первой строке прибавляем третью, умноженную на -13 , а вторую — на 8

К первой строке прибавляем четвертую, умноженную на 5/34 , ко второй прибавляем первую, умноженную на 5/17 и к третьему добавляем первое, умноженное на -3/34

По теореме Руше-Капелли система непротиворечива Независима, и решение равно



Система 10

Показать решение

Расширенная матрица системы

Примечание: Эта система была включена в объективно отметить, что теория матрицы применима к комплексным числам. Единственное отличие от предыдущих систем в том, что теперь нам нужно действовать путем умножения и деления комплексные числа.

Выполняем элементарные операции в строках для получения приведенной формы эшелона строк

Умножаем вторую строку на 1/2 и меняем ее на первую

Складываем вторую строку с первой, умноженной на — (1 + i)

Вторую строку умножаем на

Добавляем первую строку ко второй, умножаем на -i / 2

Эта последняя матрица имеет сокращенную форму, поскольку она является единичной матрицей.

По теореме Руше-Капелли система непротиворечива Независима, и решение равно



Matesfacil.com от J. Llopis под лицензией творческий Международная лицензия Commons Attribution-NonCommercial 4. 0.


7.3 — Метод исключения

7.3 — Метод исключения

7.3 — Метод исключения

Щелкните здесь, чтобы получить инструкции по использованию Algebra Coach провести метод исключения.

В этом разделе мы объясняем метод исключения. Этот метод использует тот факт, что решение уравнения не меняется, если мы

  • умножьте обе части уравнения на один и тот же коэффициент.
  • вычесть равные величины из обеих частей уравнения.
Это означает, что мы можем взять одно уравнение и вычесть несколько из другого. уравнение из него без изменения решения уравнений.

Метод исключения использует этот факт для решения системы линейных уравнений. Предположим, мы начнем с системы из n уравнений с n неизвестными. Выберите первое уравнение и вычтите из другого подходящие числа, кратные ему. n — 1 уравнение. В каждом случае выбирается кратное так что вычитание отменяет или исключает ту же переменную , скажем x . В результате уравнения n — 1 содержат только n — 1 неизвестное ( x больше не отображается).

Мы повторяем этот процесс исключения, пока не получим 1 уравнение из 1 неизвестных, которое затем легко решается.

Последний шаг — обратная замена уже полученного решения на 1 неизвестно в предыдущие уравнения, чтобы найти значения всех остальных неизвестных.



Пример: Решите эту систему уравнений методом исключения:
Решение: Возьмем дважды первое уравнение, а именно:
2 x + 2 y = 8
и вычтите его из второго уравнения, например:
В результате получается одно уравнение из одной неизвестной: и .Другой неизвестный, x , был устранены. Решение этого уравнения дает y = 0,4.

Осталось найти х . Если мы обратно подставим y = 0,4 в любое из исходных уравнений получаем х = 3,6. Таким образом, решение:

{ x = 3,6, y = 0,4}.
(Обратите внимание, что мы могли бы вместо этого найти x без обратной подстановки, если бы мы вычитали 3 раза первое уравнение из второго, так как это исключает y .)

Расширенная матрица

Мы объяснили суть исключения. Для более крупных систем нам понадобится систематическая процедура во избежание путаницы. Две такие процедуры — это исключение Гаусса и исключение Гаусса-Жордана.

Прежде чем мы их опишем, приведем краткую информацию. Система уравнений, такая как:

будет представлен прямоугольным массивом чисел, называемым Расширенная матрица :

Определения:

  • Отдельные числа в матрице называются элементами .
  • Столбцы идут вниз по матрице. например столбец 4 содержит элементы 80, 7 и 22.
  • Ряды переходят поперек. Строка 3 -го содержит 3, −1, 2 и 22. Обратите внимание, что количество столбцов в расширенной матрице всегда на 1 больше. чем количество строк.
  • Диагональ — это набор элементов, который начинается сверху, слева. угол матрицы и проходит по диагонали вниз и вправо.Диагональ указанной выше матрицы состоит из чисел 4, 1 и 2.
  • Любые числа в позиции D считаются по диагонали , любые в позиции и на выше диагонали , а любые в позиции b на меньше диагонали .
Помните следующее:
  • Строка i расширенной матрицы представляет собой i -е уравнение.
  • Столбец j (слева от вертикальной линии) содержит коэффициенты j -й переменной или неизвестного значения.
  • Вертикальная линия представляет знаки равенства.
  • Столбец справа от вертикальной линии представляет правую часть уравнений.



Элементарные операции со строками

Мы видели, что решение системы уравнений не изменится, если мы:
  • разделить обе части уравнения на константу, или
  • вычесть кратное одного уравнения из другого уравнения.
Эти же операции можно применить к строкам расширенной матрицы, поскольку каждая строка представляет собой просто уравнение. Затем они называются Операции элементарной строки .

Операции элементарной строки (E.R.O.):
  • E.R.O. # 1: Выберите строку расширенной матрицы и разделите (каждый элемент) строки константой.
  • E.R.O. # 2: Выберите любую строку расширенной матрицы и вычтите кратно любой другой строке из него (элемент за элементом).
  • E. R.O. # 3: Иногда бывает полезно поменять местами две строки. Это допустимая операция, потому что порядок уравнений несущественный.
Вы можете применять эти E.R.O. к расширенной матрице сколь угодно часто. без изменения решения уравнений, представленных матрицей.

Пример: В этом примере показано, как мы применяем E.R.O. № 1 и обозначение мы используем для обозначения этого. Разделим первую строку дополненной матрица слева на 2, чтобы получить новую расширенную матрицу справа:

Примечание: ← ÷ на 2 означает « разделить строку указывается цифрой 2 для создания новой матрицы ”.

Пример: В этом примере показано, как мы применяем E.R.O. # 2 и обозначения мы используем для обозначения этого. В расширенной матрице слева мы будем возьмите вторую строку и вычтите из нее 3 раза первую строку, чтобы получить новая расширенная матрица справа:
Примечание: R 2 — 3 · R 1 означает « возьмите строку, на которую указывает (строка 2), и вычтите 3 раза строку 1 из для создания новой строки 2.

Пример: В этом примере показано, как мы применяем E.R.O. # 3 и обозначения мы используем для обозначения этого. В расширенной матрице слева мы меняем местами строки 1 и 2, чтобы получить новая расширенная матрица справа:
Примечание: R 1 R 2 означает « поменять местами строки 1 и 2. »

Исключение по Гауссу

Процедура исключения по Гауссу представляет собой определенную последовательность E.R.O. который преобразует расширенную матрицу в форму Гаусса (также известная как эшелонированная форма ). Эта форма характеризуется единицей на диагонали и нулями под диагональю. и любые числа выше диагонали.Вот пример:
Эта расширенная матрица представляет система уравнений:
Решается обратной подстановкой. Подключение z = 3 к второе уравнение дает y = 5. Затем вставляем как z = 3, так и y = 5 в первом уравнении дает x = 7.

Алгоритм исключения по Гауссу

Мы преобразуем один столбец за раз в форму эшелона строк (или гаусса).Преобразуемый в настоящее время столбец называется поворотная колонна . Мы действуем планомерно, позволяя опорной колонне быть первым столбцом, затем вторым столбцом и т. д. до последнего столбца перед вертикальной чертой расширенной матрицы. Для каждого сводного столбца мы делаем следующие два шага, прежде чем перейти к следующему сводному столбцу:

  1. Помещаем диагональный элемент в опорную колонку. Этот элемент называется pivot .Строка, содержащая ось называется осевой строкой . Мы делим каждый элемент в сводной строке на pivot (т.е. мы используем E.R.O. # 1), чтобы получить новую строку pivot с 1 в поворотное положение.
  2. Мы получаем 0 в каждой позиции ниже позиции поворота на вычитание подходящего кратного числа сводной строки из каждого строки под ним (т. е. используя E.R.O. # 2).
Когда все столбцы слева от вертикальной линии были преобразованы с помощью процедура исключения Гаусса расширенная матрица будет в форме Гаусса и затем мы решаем систему обратной подстановкой.

Пример: Используйте метод исключения Гаусса для решения системы уравнений:

Решение: Выполните следующую последовательность запросов на добавленную стоимость в расширенной матрице:

Установите столбец вращения на столбец 1. Получите 1 в диагональном положении (красным):


Затем установите 0 под точкой поворота (красным):

Теперь пусть сводный столбец = второй столбец.

Сначала получите 1 по диагонали:


Затем получите 0 в позиции под точкой поворота:

Теперь пусть сводный столбец = третий столбец. Получите 1 по диагонали:

Эта матрица, которая теперь имеет форму Гаусса, представляет собой систему трех уравнения:

Решается обратной подстановкой. Вставляем z = 3 во вторую уравнение получаем y = 5.И вставка z = 3 и y = 5 в первое уравнение получаем x = 7. Таким образом, решение следующее:
{ x = 7, y = 5, z = 3}.


Гаусс-Джордан Ликвидация

Процедура исключения Гаусса-Джордана представляет собой несколько иную последовательность E.R.O., который преобразует расширенную матрицу в форму Гаусса-Жордана (также известная как сокращенная форма эшелона ).Эта форма характеризуется единицей на диагонали, 0 над и под диагональю слева от вертикальной линии, и любые числа справа от вертикальной линии. Вот пример:
Эта расширенная матрица представляет собой систему уравнений:
Эта система уже решена: x = 7, y = 5, z = 3. Обратной замены не требуется. Однако примерно в два раза больше E.R.O. требуются для производства Форма Гаусса-Жордана как форма Гаусса.

Алгоритм исключения Гаусса-Жордана

Мы преобразуем по одному столбцу за раз в сокращенную форму эшелона строк (или форму Гаусса-Жордана). Преобразуемый в настоящее время столбец называется поворотная колонна . Мы действуем планомерно, позволяя опорной колонне быть первым столбцом, затем вторым столбцом и т. д. до последнего столбца перед вертикальной чертой расширенной матрицы.Для каждого сводного столбца мы делаем следующие два шага, прежде чем перейти к следующему сводному столбцу:

  1. Помещаем диагональный элемент в опорную колонку. Этот элемент называется pivot . Строка, содержащая ось называется осевой строкой . Мы делим каждый элемент в сводной строке на pivot (т.е. мы используем E.R.O. # 1), чтобы получить новую строку pivot с 1 в поворотное положение.
  2. Мы получаем 0 в каждой позиции выше и ниже позиции поворота на вычитание подходящего кратного числа сводной строки из каждого ряды над и под ним (т.е. с помощью E.R.O. №2).
Когда все столбцы слева от вертикальной линии будут преобразованы, расширенная матрица будет в форме Гаусса-Жордана, и мы сможем считать решение из правого столбца.

Обратите внимание, что единственное отличие от процедуры Гаусса состоит в том, что на втором этапе мы получаем 0s выше диагонали , а также ниже диагонали. Важно получить все эти нули, прежде чем переходить к следующему сводному столбцу.

Пример: Используйте метод исключения Гаусса-Жордана для решения системы уравнений:
Решение: Выполните эту последовательность E.R.O. на расширенной матрице. Установите для сводного столбца столбец 1. Получите 1 по диагонали. (красным) с помощью E.R.O. №1:
Затем установите 0 под точкой поворота (красным) с помощью E.R.O. # 2:
Теперь пусть сводный столбец = второй столбец.Сначала получите 1 по диагонали. с помощью E.R.O. №1:
Затем получите 0 в позициях выше и ниже точки поворота (красным) с помощью E.R.O. # 2:
Теперь пусть сводный столбец = третий столбец. Получите 1 в диагональном положении, используя E.R.O. №1:
Затем получите 0 в позициях над точкой поворота (красным) с помощью E.R.O. # 2:
Эта матрица, которая теперь имеет форму Гаусса-Жордана или сокращенную форму эшелона строк, представляет собой решение:
{ x = 49, y = −18, z = 8}.



Резервные и несовместимые системы

Если количество уравнений больше, чем количество неизвестных, то системы гарантированно будут либо избыточными или непоследовательны. Но если количество уравнения равно или меньше количества неизвестных, тогда вы, как правило, не распознавать систему как избыточную или несовместимую до самого конца расчета. Это особенно актуально, если система большая.

Если вы решаете систему уравнений методом подстановки и система избыточна, тогда вы получите окончательное уравнение, в котором 0 = 0. Или, если система несовместима, вы получите такую, которая утверждает противоречие вида 0 = 5. Нечто подобное происходит при использовании исключения Гаусса или Гаусса-Джордана. Если система резервная, то по окончании процедуры устранения когда у вас есть расширенная матрица в форме Гаусса или Гаусса-Жордана, последняя строка расширенной матрицы будет:

Эта последняя строка представляет уравнение 0 = 0, бесполезную информацию.

Если система несовместима, то последняя строка расширенной матрицы будет выглядят примерно так:

Последняя строка представляет уравнение 0 = 5, противоречие. Попробуйте упражнения, которые содержат примеры избыточные и противоречивые системы уравнений.

Пример: Используйте метод исключения Гаусса, чтобы преобразовать эту систему уравнений в форму эшелона строк и интерпретировать результат:
Решение: Выполните эту последовательность E.R.O. на расширенной матрице. Установите столбец поворота на столбец 1. В позиции поворота уже стоит 1, поэтому переходите к получению 0 под поворотом:
Теперь установите сводный столбец во второй столбец. В позиции поворота уже есть 1, поэтому переходите к получению 0 под точкой поворота:
Теперь установите сводный столбец в третий столбец. Первое, что нужно сделать, это получить 1 по диагонали, но нет способа сделать это. Фактически эта матрица уже представлена ​​в виде эшелона строк и представляет:
Эта система уравнений противоречива и не имеет решения. Последнее уравнение утверждает противоречие, а именно 0 = −50.
В общем, расширенная матрица, которая была преобразована в форму эшелона строк и которая содержит одну или несколько нижних строк, состоящих из всех нулей слева от вертикальной линии и ненулевого числа справа, указывает на несовместимую систему уравнения без решения.



Уравнений меньше, чем неизвестных

Если количество уравнений в системе меньше количества неизвестных, то вы достигнете точки Гаусса или Гаусса-Иордана процедура, в которой вы не можете преобразовать стержень столбец, потому что у вас закончились сводные строки.Вот пример:
В третьем столбце нет поворота и строки поворота, поэтому вам нужно остановиться. Эта расширенная матрица представляет эту систему уравнений:
Во второй форме мы видим, что если задано значение для z , то x и y можно выразить через него. Следующая матрица показывает, что предоставление значения для z , скажем z = 5, составляет другую строку:
Попробуйте упражнения, содержащие примеры систем с меньшим количеством уравнений. чем неизвестные.



Если вы нашли эту страницу в поиске в Интернете, вы не увидите
Оглавление в рамке слева.
Щелкните здесь, чтобы отобразить его.

Как выполнить исключение Гаусса? Шкаф системы с уникальным решением

В этом разделе мы предлагаем еще один пример решения системы линейных алгебраических уравнений методом исключения Гаусса. Этот пример ясно показывает, что при выполнении исключения Гаусса вы должны замечать, когда удобно поменять местами строки, чтобы сэкономить время и сократить вычисления.

Нам предстоит решить следующую систему линейных алгебраических уравнений:

\ left \ {\ begin {align} -2x + y + 2z = 2 \\ x + 3y-z = 0 \\ 3x + 2y + 4z = 3 \ end {align} \ right.

Прежде всего, обратите внимание, что мы можем поменять местами любые два уравнения в системе по своему усмотрению, решение, очевидно, не изменится. Мы видим, что x имеет коэффициент 1 во втором уравнении, поэтому мы можем преобразовать нашу систему следующим образом:

\ left \ {\ begin {align} x + 3y-z = 0 \\ — 2x + y + 2z = 2 \\ 3x + 2y + 4z = 3 \ end {align} \ right.

Вот видеоверсия этого примера:

Теперь мы можем исключить x из второго уравнения.Для этого добавим первое уравнение, умноженное на 2, ко второму:

\ left \ {\ begin {align} x + 3y-z = 0 \\ 7y = 2 \\ 3x + 2y + 4z = 3 \ end {align} \ right.

Также мы исключаем x из третьего уравнения: вычитаем первое уравнение, умноженное на 3, из третьего:

\ left \ {\ begin {align} x + 3y-z = 0 \\ 7y = 2 \\ — 7y + 7z = 3 \ end {align} \ right.

После этого мы могли действовать по нашему алгоритму. Но в этом нет необходимости, поскольку мы видим, что у нас есть второе уравнение, содержащее только одно неизвестное y, а также третье уравнение, содержащее две неизвестные.Таким образом, мы можем изменить нашу систему следующим образом:

\ left \ {\ begin {выравнивается} x + 3y-z = 0 \\ — 7y + 7z = 3 \\ 7y = 2 \ end {выравнивается} \ right.

Это треугольная форма; мы можем сразу получить y из третьего уравнения:

\ left \ {\ begin {align} x + 3y-z = 0 \\ — 7y + 7z = 3 \\ y = \ frac {2} {7} \ end {align} \ right.

Затем мы подставляем его во второе уравнение и получаем значение z:

\ left \ {\ begin {align} x + 3y-z = 0 \\ z = \ frac {5} {7} \\ y = \ frac {2} {7} \ end {align} \ right.

Затем подставьте значения y, z в первое уравнение, чтобы получить x:

\ left \ {\ begin {выравнивается} x = — \ frac {1} {7} \\ z = \ frac {5} {7} \\ y = \ frac {2} {7} \ end {выравнивается} \правильно.

и вот ответ.

Как видите, у данной системы есть одно решение. Это решение уникальное. Имейте в виду, что в общем случае есть три возможных случая: система линейных алгебраических уравнений либо имеет единственное решение, либо бесконечно много решений, либо вообще не имеет решения. Решая домашнее задание по линейной алгебре, вы можете столкнуться с любым из этих случаев. Не забудьте потом проверить свой ответ. Просто подставьте его в исходную систему. Также вы можете использовать онлайн-решатель систем.Введите свою систему и сравните результат со своим ответом.

Ознакомьтесь с подробными объяснениями метода исключения Гаусса в одном из наших предыдущих руководств для лучшего понимания. Помните, что практика — ключевой момент в изучении математики. Хотите лучше по математике? Уделите больше математики!

.

Добавить комментарий

Ваш адрес email не будет опубликован. Обязательные поля помечены *